Rocksolid Light

Welcome to novaBBS (click a section below)

mail  files  register  newsreader  groups  login

Message-ID:  

19 May, 2024: Line wrapping has been changed to be consistent with Usenet standards.
 If you find that it is broken please let me know here rocksolid.nodes.help


tech / sci.physics.relativity / Re: If time goes slower for each twin

SubjectAuthor
* If time goes slower for each twinmitchr...@gmail.com
+* Re: If time goes slower for each twinRichard Hachel
|+* Re: If time goes slower for each twinSylvia Else
||`* Re: If time goes slower for each twinRichard Hachel
|| `* Re: If time goes slower for each twinJ. J. Lodder
||  `- Re: If time goes slower for each twinRichard Hachel
|`* Re: If time goes slower for each twinWerner Dryer
| `* Re: If time goes slower for each twinRichard Hachel
|  +* Re: If time goes slower for each twinSylvia Else
|  |+* Re: If time goes slower for each twinRichard Hachel
|  ||+* Re: If time goes slower for each twinSylvia Else
|  |||`- Re: If time goes slower for each twinRichard Hachel
|  ||+- Re: If time goes slower for each twinThomas 'PointedEars' Lahn
|  ||`* Re: If time goes slower for each twinPaul B. Andersen
|  || +- Re: If time goes slower for each twinMaciej Wozniak
|  || `- Re: If time goes slower for each twinRichard Hachel
|  |`- Re: If time goes slower for each twinrotchm
|  +- Re: If time goes slower for each twinThomas 'PointedEars' Lahn
|  `* Re: If time goes slower for each twinVidal Rhum
|   `* Re: If time goes slower for each twinRichard Hachel
|    +* Re: If time goes slower for each twinVidal Rhum
|    |`- Re: If time goes slower for each twinRichard Hachel
|    `* Re: If time goes slower for each twinThomas 'PointedEars' Lahn
|     `* Re: If time goes slower for each twinRichard Hachel
|      `* Re: If time goes slower for each twinRichard Hachel
|       +* Re: If time goes slower for each twinThomas 'PointedEars' Lahn
|       |+* Re: If time goes slower for each twinRichard Hachel
|       ||`* Re: If time goes slower for each twinThomas 'PointedEars' Lahn
|       || +* Re: If time goes slower for each twinRichard Hachel
|       || |+* Re: If time goes slower for each twinmitchr...@gmail.com
|       || ||`* Re: If time goes slower for each twinRichard Hachel
|       || || `* Re: If time goes slower for each twinOdd Bodkin
|       || ||  `* Re: If time goes slower for each twinMaciej Wozniak
|       || ||   +* Re: If time goes slower for each twinOdd Bodkin
|       || ||   |`* Re: If time goes slower for each twinMaciej Wozniak
|       || ||   | +* Re: If time goes slower for each twinJ. J. Lodder
|       || ||   | |+* Re: If time goes slower for each twinMaciej Wozniak
|       || ||   | ||`* Re: If time goes slower for each twinJ. J. Lodder
|       || ||   | || `* Re: If time goes slower for each twinMaciej Wozniak
|       || ||   | ||  `- Re: If time goes slower for each twinJ. J. Lodder
|       || ||   | |`* Re: If time goes slower for each twinThomas 'PointedEars' Lahn
|       || ||   | | +- Re: If time goes slower for each twinMaciej Wozniak
|       || ||   | | `* Re: If time goes slower for each twinJ. J. Lodder
|       || ||   | |  +- Re: If time goes slower for each twinThomas 'PointedEars' Lahn
|       || ||   | |  `* Re: If time goes slower for each twinThomas 'PointedEars' Lahn
|       || ||   | |   `* Re: If time goes slower for each twinJ. J. Lodder
|       || ||   | |    +* Re: If time goes slower for each twinMaciej Wozniak
|       || ||   | |    |`* Re: If time goes slower for each twinJ. J. Lodder
|       || ||   | |    | `* Re: If time goes slower for each twinMaciej Wozniak
|       || ||   | |    |  `* Re: If time goes slower for each twinJ. J. Lodder
|       || ||   | |    |   `* Re: If time goes slower for each twinMaciej Wozniak
|       || ||   | |    |    `* Re: If time goes slower for each twinJ. J. Lodder
|       || ||   | |    |     +* Re: If time goes slower for each twinMaciej Wozniak
|       || ||   | |    |     |`* Re: If time goes slower for each twinJ. J. Lodder
|       || ||   | |    |     | `- Re: If time goes slower for each twinMaciej Wozniak
|       || ||   | |    |     +* Re: If time goes slower for each twinTom Roberts
|       || ||   | |    |     |+* Re: If time goes slower for each twinJ. J. Lodder
|       || ||   | |    |     ||+- Re: If time goes slower for each twinMaciej Wozniak
|       || ||   | |    |     ||`* Re: If time goes slower for each twinTom Roberts
|       || ||   | |    |     || +- Re: If time goes slower for each twinMaciej Wozniak
|       || ||   | |    |     || `* Re: If time goes slower for each twinJ. J. Lodder
|       || ||   | |    |     ||  +* Re: If time goes slower for each twinMichael Moroney
|       || ||   | |    |     ||  |`* Re: If time goes slower for each twinTom Roberts
|       || ||   | |    |     ||  | `* Re: If time goes slower for each twinJ. J. Lodder
|       || ||   | |    |     ||  |  `- Re: If time goes slower for each twinMaciej Wozniak
|       || ||   | |    |     ||  +* Re: If time goes slower for each twinTom Roberts
|       || ||   | |    |     ||  |+- Re: If time goes slower for each twinMaciej Wozniak
|       || ||   | |    |     ||  |`- Re: If time goes slower for each twinmitchr...@gmail.com
|       || ||   | |    |     ||  `* Re: If time goes slower for each twinMaciej Wozniak
|       || ||   | |    |     ||   `- Re: If time goes slower for each twinJ. J. Lodder
|       || ||   | |    |     |`- Re: If time goes slower for each twinVaugn Rhea
|       || ||   | |    |     `- Re: If time goes slower for each twinMaciej Wozniak
|       || ||   | |    `* Re: If time goes slower for each twinRichD
|       || ||   | |     +- Re: If time goes slower for each twinmitchr...@gmail.com
|       || ||   | |     +* Re: If time goes slower for each twinThomas Heger
|       || ||   | |     |+* Re: If time goes slower for each twinwhodat
|       || ||   | |     ||`- Re: If time goes slower for each twinThomas Heger
|       || ||   | |     |`- Re: If time goes slower for each twinmitchr...@gmail.com
|       || ||   | |     `* Re: If time goes slower for each twinJ. J. Lodder
|       || ||   | |      +* Re: If time goes slower for each twinTom Roberts
|       || ||   | |      |`* Re: If time goes slower for each twinJ. J. Lodder
|       || ||   | |      | +- Re: If time goes slower for each twinMaciej Wozniak
|       || ||   | |      | +- Re: If time goes slower for each twinPaparios
|       || ||   | |      | `- Re: If time goes slower for each twinMaciej Wozniak
|       || ||   | |      `* Re: If time goes slower for each twinMark-T
|       || ||   | |       +* Re: If time goes slower for each twinOdd Bodkin
|       || ||   | |       |+- Re: If time goes slower for each twinMaciej Wozniak
|       || ||   | |       |`* Re: If time goes slower for each twinMark-T
|       || ||   | |       | +- Re: If time goes slower for each twinJ. J. Lodder
|       || ||   | |       | `* Re: If time goes slower for each twinOdd Bodkin
|       || ||   | |       |  `* Re: If time goes slower for each twinJ. J. Lodder
|       || ||   | |       |   `* Re: If time goes slower for each twinMaciej Wozniak
|       || ||   | |       |    `- Re: If time goes slower for each twinJ. J. Lodder
|       || ||   | |       `- Re: If time goes slower for each twinTom Roberts
|       || ||   | `* Re: If time goes slower for each twinOdd Bodkin
|       || ||   |  `* Re: If time goes slower for each twinMaciej Wozniak
|       || ||   |   `* Re: If time goes slower for each twinOdd Bodkin
|       || ||   |    `* Re: If time goes slower for each twinMaciej Wozniak
|       || ||   |     `* Re: If time goes slower for each twinOdd Bodkin
|       || ||   |      +- Re: If time goes slower for each twinMaciej Wozniak
|       || ||   |      `* Re: If time goes slower for each twinJ. J. Lodder
|       || ||   `- Re: If time goes slower for each twinRichard Hachel
|       || |+* Re: If time goes slower for each twinThomas 'PointedEars' Lahn
|       || |`- Re: If time goes slower for each twinOdd Bodkin
|       || `- Re: If time goes slower for each twinMaciej Wozniak
|       |`- Re: If time goes slower for each twinMaciej Wozniak
|       +* Re: If time goes slower for each twinOdd Bodkin
|       `* Re: If time goes slower for each twinPaul B. Andersen
+- Re: If time goes slower for each twinWerner Dryer
+- Re: If time goes slower for each twinThomas 'PointedEars' Lahn
+* Re: If time goes slower for each twinJ. J. Lodder
+* Re: If time goes slower for each twinKen Seto
+* Re: If time goes slower for each twinThe Starmaker
+* Re: If time goes slower for each twinKen Seto
`* Re: If time goes slower for each twineverything isalllies

Pages:12345678910111213141516
Re: If time goes slower for each twin

<1pkfhrc.aw7su3eyz58rN%nospam@de-ster.demon.nl>

  copy mid

https://www.novabbs.com/tech/article-flat.php?id=74896&group=sci.physics.relativity#74896

  copy link   Newsgroups: sci.physics.relativity
Path: i2pn2.org!i2pn.org!eternal-september.org!reader02.eternal-september.org!.POSTED!not-for-mail
From: nos...@de-ster.demon.nl (J. J. Lodder)
Newsgroups: sci.physics.relativity
Subject: Re: If time goes slower for each twin
Date: Sun, 19 Dec 2021 22:28:59 +0100
Organization: De Ster
Lines: 38
Message-ID: <1pkfhrc.aw7su3eyz58rN%nospam@de-ster.demon.nl>
References: <edb7909e-3fb6-40aa-b919-d164417c4383n@googlegroups.com> <sp7mlj$1kmk$1@gioia.aioe.org> <ef48c7ff-2589-4eba-8328-243a3402af47n@googlegroups.com> <1pk4xfm.9ayruk1i0tv8vN%nospam@de-ster.demon.nl> <8868015.CDJkKcVGEf@PointedEars.de> <1pk6cuj.1mz43ke4p1ilN%nospam@de-ster.demon.nl> <2223314.ElGaqSPkdT@PointedEars.de> <1pk8wy0.1hwtxny6rumisN%nospam@de-ster.demon.nl> <4489c1ea-05b3-43db-847a-a3644ce73c29n@googlegroups.com> <1pk94f3.1qgx6jp1ceyeocN%nospam@de-ster.demon.nl> <b246cde1-e36d-4448-8cad-e2d55a813018n@googlegroups.com> <1pk9z2x.1ncbcseon2cpbN%nospam@de-ster.demon.nl> <68bfdd09-e167-4e49-ba51-6b301e595190n@googlegroups.com> <1pk9zu9.14hdv7x1414yhoN%nospam@de-ster.demon.nl> <afOdnU7OHavWASb8nZ2dnUU7_83NnZ2d@giganews.com> <1pkauap.lnk9ul1adhmndN%nospam@de-ster.demon.nl> <OaudnSddyY_D5yD8nZ2dnUU7_8zNnZ2d@giganews.com> <1pkdq1a.1iq87uqnm3fmvN%nospam@de-ster.demon.nl> <spm5mi$363$1@gioia.aioe.org> <TeSdncTdEok_JSP8nZ2dnUU7_83NnZ2d@giganews.com>
Reply-To: jjlax32@xs4all.nl (J. J. Lodder)
Injection-Info: reader02.eternal-september.org; posting-host="d6dfe58064eb127e906d52b2103683fc";
logging-data="23083"; mail-complaints-to="abuse@eternal-september.org"; posting-account="U2FsdGVkX19rFyTT0S66GY9JUtADLCL4ekePNgeREt4="
User-Agent: MacSOUP/2.8.5 (ea919cf118) (Mac OS 10.10.5)
Cancel-Lock: sha1:kEx9FBZ66epfSmvGgKEq2DvwlOQ=
 by: J. J. Lodder - Sun, 19 Dec 2021 21:28 UTC

Tom Roberts <tjroberts137@sbcglobal.net> wrote:

> On 12/18/21 8:33 PM, Michael Moroney wrote:
> > clocks tick at their standard rate. It is only comparing signals
> > from other clocks under different circumstances where this could be
> > stated. Someone in Boulder would say the clock in Paris runs slow
> > and they'd be correct, for Boulder. Someone in Paris would say the
> > Boulder clock runs fast, and *they'd* be correct, for Paris (or
> > London etc.)
>
> No. You used incorrect English and missed the essential point. A person
> in Boulder would say the clock in Paris runs slow COMPARED TO MY CLOCK.
> A person in Paris would say the Boulder clock runs fast COMPARED TO MY
> CLOCK. And if used for scientific purposes they would describe how the
> comparisons are made.
>
> Basic English: "slow" and "fast" are comparisons [#], and here they
> necessarily relate two clocks to each other.
>
> [#] More properly, "slower" and "faster", but both forms
> are used.

Now you have nothing better left than carping about use of English?

'Slow' and 'fast' do not need a 'compared to' when there is a well
understood common standard.
For atomic clocks this is of course TAI.

But the usage is much older than that.
Phileas Fogg for example could have said:
"My watch is a minute fast" (with 'a day' understood)
when he heard the strike of the Big Ben. [1]

Jan

[1] Fogg and his fellow Londeners would know that Big Ben
was kept to within a second of the true GMT.

Re: If time goes slower for each twin

<1pkfhts.1dzlthd1upihk2N%nospam@de-ster.demon.nl>

  copy mid

https://www.novabbs.com/tech/article-flat.php?id=74897&group=sci.physics.relativity#74897

  copy link   Newsgroups: sci.physics.relativity
Path: i2pn2.org!i2pn.org!eternal-september.org!reader02.eternal-september.org!.POSTED!not-for-mail
From: nos...@de-ster.demon.nl (J. J. Lodder)
Newsgroups: sci.physics.relativity
Subject: Re: If time goes slower for each twin
Date: Sun, 19 Dec 2021 22:29:00 +0100
Organization: De Ster
Lines: 42
Message-ID: <1pkfhts.1dzlthd1upihk2N%nospam@de-ster.demon.nl>
References: <uwTWBSS2m6_vwEEBIi38LzTd43M@jntp> <sp3frr$itm$1@gioia.aioe.org> <VuJDUZEFa_vj-RHzwrdu0oE2wvc@jntp> <sp4mus$78a$1@gioia.aioe.org> <O90D20WUu984acPU1w_S90qqcuU@jntp> <4676671.GXAFRqVoOG@PointedEars.de> <6MmCLEceSpwh8yfl41Jlt8j_tlA@jntp> <UXW1-e65QWxhFcysGp_wEQcIDvM@jntp> <2089212.irdbgypaU6@PointedEars.de> <EQ3l7crtJC_TXLNyfniK_mjBZSo@jntp> <12914471.uLZWGnKmhe@PointedEars.de> <RHSgLwi_4mlmyKU2Qx7KMAUkVkY@jntp> <03e8378e-c4e4-469b-9316-15e3d9d5aab1n@googlegroups.com> <v-Z96JM7oeo3z2tAH3OacbwhD2E@jntp> <sp7jnu$200$3@gioia.aioe.org> <edb7909e-3fb6-40aa-b919-d164417c4383n@googlegroups.com> <sp7mlj$1kmk$1@gioia.aioe.org> <ef48c7ff-2589-4eba-8328-243a3402af47n@googlegroups.com> <1pk4xfm.9ayruk1i0tv8vN%nospam@de-ster.demon.nl> <8868015.CDJkKcVGEf@PointedEars.de> <1pk6cuj.1mz43ke4p1ilN%nospam@de-ster.demon.nl> <2223314.ElGaqSPkdT@PointedEars.de> <1pk8wy0.1hwtxny6rumisN%nospam@de-ster.demon.nl> <4c77b5bc-722f-4fc7-aae6-43e7663b1569n@googlegroups.com>
Reply-To: jjlax32@xs4all.nl (J. J. Lodder)
Injection-Info: reader02.eternal-september.org; posting-host="d6dfe58064eb127e906d52b2103683fc";
logging-data="23083"; mail-complaints-to="abuse@eternal-september.org"; posting-account="U2FsdGVkX1+DGc7Zpx8rD63CTCxiAaj9Ktb3vmGIsY8="
User-Agent: MacSOUP/2.8.5 (ea919cf118) (Mac OS 10.10.5)
Cancel-Lock: sha1:/jQ/J77kjiyiWaH9m11LkkFq2CU=
 by: J. J. Lodder - Sun, 19 Dec 2021 21:29 UTC

RichD <r_delaney2001@yahoo.com> wrote:

> On December 15, J. J. Lodder wrote:
> >>> Certainly, each clock runs at its own rate in its own local rest frame.
>
> Define "runs at its own rate"
>
> >> But the reason is NOT that one clock is ticking faster than the other, but
> >> that one clock measures *more elapsed proper time* than the other.
> >
> > There is nothing relative about it: clocks that stand higher run faster,
> > objectively, absolutely, and for all observers.
>
> Define "runs faster"

Do you really need that?
It runs faster if its ticks fall behind.

| | | | |
| | | | |

Add 10^16 spaces for yourself.
(and buy a bigger screen)

> Let's try to operationalize the topic. Given two space stations, in orbit
> around earth, one at 1000 miles altitude, the other at 2000 miles. Each
> is a satellite, in free fall.
>
> They carry identical clocks, and a cubic meter of ice. They time the ice
> cube melt. Per your claim above, the higher clock runs faster. Therefore,
> it counts more clock ticks until the cube melts. Agreed?

Free falling observers see the same laws of physics,
in their rest frame.

> Tom Roberts engages in the same verbalisms; i.e. "clocks run at their own
> rate" It's vacuous, tautologous.

Yes, by postulate. I already said so in a previous post.

Jan

Re: If time goes slower for each twin

<1pkfhui.3xn22u16ilj48N%nospam@de-ster.demon.nl>

  copy mid

https://www.novabbs.com/tech/article-flat.php?id=74898&group=sci.physics.relativity#74898

  copy link   Newsgroups: sci.physics.relativity
Path: i2pn2.org!i2pn.org!eternal-september.org!reader02.eternal-september.org!.POSTED!not-for-mail
From: nos...@de-ster.demon.nl (J. J. Lodder)
Newsgroups: sci.physics.relativity
Subject: Re: If time goes slower for each twin
Date: Sun, 19 Dec 2021 22:29:00 +0100
Organization: De Ster
Lines: 59
Message-ID: <1pkfhui.3xn22u16ilj48N%nospam@de-ster.demon.nl>
References: <ade8707c-5779-44d2-9599-c42b4c2cdbf3n@googlegroups.com> <1pk4fg7.6nqt0j3osf61N%nospam@de-ster.demon.nl> <2088029.irdbgypaU6@PointedEars.de> <1pk7zpw.1u4709u12ez725N%nospam@de-ster.demon.nl> <jdadncq2e-PC5Cf8nZ2dnUU7_83NnZ2d@giganews.com> <1pk9vhw.133tjv1sel3x2N%nospam@de-ster.demon.nl> <ec11547f-e187-4a95-96f4-2bb1d4a309b4n@googlegroups.com> <1pkac7z.1frr3nrtaz7ilN%nospam@de-ster.demon.nl> <nK6dnY3H75YdDib8nZ2dnUU7_83NnZ2d@giganews.com> <1pkell2.1h3cfauu373o3N%nospam@de-ster.demon.nl> <86f37c21-821d-4d66-9e30-8f487644bb62n@googlegroups.com>
Reply-To: jjlax32@xs4all.nl (J. J. Lodder)
Mime-Version: 1.0
Content-Type: text/plain; charset=ISO-8859-1
Content-Transfer-Encoding: 8bit
Injection-Info: reader02.eternal-september.org; posting-host="d6dfe58064eb127e906d52b2103683fc";
logging-data="23083"; mail-complaints-to="abuse@eternal-september.org"; posting-account="U2FsdGVkX1/lHpBeUJ+LQrLUHccr6AhC2aR3sTRvEdw="
User-Agent: MacSOUP/2.8.5 (ea919cf118) (Mac OS 10.10.5)
Cancel-Lock: sha1:G9QenD9T7MzG0hg7mOdbJ/aDQXM=
 by: J. J. Lodder - Sun, 19 Dec 2021 21:29 UTC

Paparios <mrios@ing.puc.cl> wrote:

> El sábado, 18 de diciembre de 2021 a las 19:32:21 UTC-3, J. J. Lodder:
> > Tom Roberts <tjrobe...@sbcglobal.net> wrote:
> >
> > > On 12/16/21 8:42 AM, J. J. Lodder wrote:
> > > > Gravitational time dilatation is not a propagation effect.
> > >
> > > Hmmmm. See below.
> > >
> > > > It is inherent in the gravitational potential being higher or lower.
> > > > (yes, in Newtonian language)
> > >
> > > Hmmmm. See below.
> > You should stop Hmmm-Hmmming, and think and speak clearly.
> >
> > To make sure we talk about the same things I give a simple experiment.
> > Suppose we have a lab, and several optical clocks.
> > There is one on the lab bench, and suppose for convenience
> > that it ticks at 500 THz precisely, so at 5*10^14 Hz.
> > (with a stability of 10^-18)
> > Now take a second identical clock,
> > and place it on a pedestal about one meter higher on the bench.
> > So with a \Delta\Phi of 10 m^2/s^2. (with c^2 ~= 10^17)
> >
>
> That one meter difference in height, means both clocks are following
> different paths through spacetime.

While being both at rest in your room.
With sufficient purism you can even insist
that no two atoms can follow the same path through space-time.
That is just silly pedantry, without any empirical meaning.

> > By everybody's understanding of general relativity
> > (and perhaps also by yours)
> > it will tick at a rate of (1 + 10^-16) THz,
> > so at 5 x 10^14 + 0.05 Hz.
> > This difference in frequency is easily demonstrated.
> > Take light from the upper clock,
> > and bring it down with mirrors to the lower one,
> > and let them interfere.
> > You will see beats with a period of 20 seconds.
> > Bring light from the lower one up, and idem.
> > By direct frequency measurement using laser spectroscopy
> > you can establish that the higher clock is faster,
> > as predicted by general relativity.
>
> Sure, but in that case you are sending signals from the higher clock to
> the lower clock and then making the comparison there. In order to measure
> the time interval between them, the receiving clock must project that
> displacement 4-vector onto itself; that is a dot product involving the
> metric at the receiving clock's location.

No need to, but you snipped that.
In principle each clock can just count its ticks,
and you can see the counters diverge.

Jan

Re: If time goes slower for each twin

<1pkfhvn.8y3k8armm3o0N%nospam@de-ster.demon.nl>

  copy mid

https://www.novabbs.com/tech/article-flat.php?id=74899&group=sci.physics.relativity#74899

  copy link   Newsgroups: sci.physics.relativity
Path: i2pn2.org!i2pn.org!eternal-september.org!reader02.eternal-september.org!.POSTED!not-for-mail
From: nos...@de-ster.demon.nl (J. J. Lodder)
Newsgroups: sci.physics.relativity
Subject: Re: If time goes slower for each twin
Date: Sun, 19 Dec 2021 22:29:00 +0100
Organization: De Ster
Lines: 144
Message-ID: <1pkfhvn.8y3k8armm3o0N%nospam@de-ster.demon.nl>
References: <ade8707c-5779-44d2-9599-c42b4c2cdbf3n@googlegroups.com> <1pk4fg7.6nqt0j3osf61N%nospam@de-ster.demon.nl> <2088029.irdbgypaU6@PointedEars.de> <1pk7zpw.1u4709u12ez725N%nospam@de-ster.demon.nl> <jdadncq2e-PC5Cf8nZ2dnUU7_83NnZ2d@giganews.com> <1pk9vhw.133tjv1sel3x2N%nospam@de-ster.demon.nl> <ec11547f-e187-4a95-96f4-2bb1d4a309b4n@googlegroups.com> <1pkac7z.1frr3nrtaz7ilN%nospam@de-ster.demon.nl> <nK6dnY3H75YdDib8nZ2dnUU7_83NnZ2d@giganews.com> <1pkell2.1h3cfauu373o3N%nospam@de-ster.demon.nl> <t5udnfZSD4BtUCP8nZ2dnUU7_83NnZ2d@giganews.com>
Reply-To: jjlax32@xs4all.nl (J. J. Lodder)
Injection-Info: reader02.eternal-september.org; posting-host="d6dfe58064eb127e906d52b2103683fc";
logging-data="23083"; mail-complaints-to="abuse@eternal-september.org"; posting-account="U2FsdGVkX18iNW5FSoEkJ32PaNNtVXw7Cu0Cr1DhCRc="
User-Agent: MacSOUP/2.8.5 (ea919cf118) (Mac OS 10.10.5)
Cancel-Lock: sha1:P2EwbApov6EIB8jrNvb7zVGY4Ks=
 by: J. J. Lodder - Sun, 19 Dec 2021 21:29 UTC

Tom Roberts <tjroberts137@sbcglobal.net> wrote:

> On 12/18/21 4:32 PM, J. J. Lodder wrote:
> > To make sure we talk about the same things I give a simple experiment.
> > Suppose we have a lab, and several optical clocks.
> > There is one on the lab bench, and suppose for convenience
> > that it ticks at 500 THz precisely, so at 5*10^14 Hz.
> > (with a stability of 10^-18)
> > Now take a second identical clock,
> > and place it on a pedestal about one meter higher on the bench.
> > So with a \Delta\Phi of 10 m^2/s^2. (with c^2 ~= 10^17)
> >
> > By everybody's understanding of general relativity
> > (and perhaps also by yours)
> > it will tick at a rate of (1 + 10^-16) THz,
> > so at 5 x 10^14 + 0.05 Hz.
>
> This is wrong. You are presuming your own MISUNDERSTANDING of General
> Relativity. "Everybody" does not share your misunderstanding, and
> physicists who understand GR will disagree with your claim. Each clock
> will tick at 500 THz, because they are identical [#].

So you are in denial of observed reality.
> [#] I am talking about the clocks, AND ONLY THE CLOCKS.
> So their tick rate is their intrinsic tick rate, which can
> only be measured in their instantaneously co-movung locally
> inertial frame (ICLIF). In this case, since the clocks are
> so very accurate, 1 meter difference in altitude is enough
> to make each one's ICLIF too small to include the other clock.

The clocks are at rest with respect to each other,
firmly bolted to the same lab table.

But your whole ICLIF thing is a fundamental error.
The difference in clock rates is NOT caused by the local gravity.
It is not a local effect,
so considering local inertial frames can't deal with it.
If the geoid changes, so will the difference in clock rates between
Boulder and Paris change.

> > This difference in frequency is easily demonstrated.
> > Take light from the upper clock,
> > and bring it down with mirrors to the lower one,
> > and let them interfere.
> > You will see beats with a period of 20 seconds.
>
> Sure. But what is the effect of gravity on the light signal, and on the
> way it is measured? You MUST include that in your analysis.

Of course not. We just see the clocks ticking at different rates.
Purely empirical, without any need for a theory.

> GR is crystal clear: the beat is caused by the difference in the time
> component of the metric between the clock and the observing instrument;
> it appears in the geometrical projection of signal onto instrument. It
> is NOT due to "identical clocks ticking at different rates" as you think.

That's just your way of denying
that you see clocks ticking at different rates,
while standing next to them.

> > Bring light from the lower one up, and idem.
> > By direct frequency measurement using laser spectroscopy
> > you can establish that the higher clock is faster,
> > as predicted by general relativity.
>
> GR does not predict the upper clock is "faster", but it does predict the
> beat, due to the way the interference is observed.

Nice sophistery.
They differ in rate because they beat, yet they run at the same rate.
(but only in your mind's eye)

> If you are going to apply "direct frequency measurement" to these
> clocks, you need to connect each clock to a measuring instrument, and
> must include the effect of gravitation on the signal between clock and
> instrument.

You clearly don't know how real life atomic clocks work.
The counting mechanism is an inherent and integral part
of the works inside.

> If you do that by connecting each clock to an instrument
> that is not moved, then the upper clock will be measured to tick faster
> than the lower clock. If you then analyze the effect of gravity on those
> signals, and correct the instrument's readings for it, you will find
> both clocks actually tick at 500 THz [@].

No signals are needed.
Idealising: both clocks have running digits,
counting periods of the clock,
and you can see the higher one getting ahead,
just by looking at them.
(from whatever point you want to look)

> [@] Again I remind you that this English phrase refers
> just to the clocks, so it refers to their intrinsic
> tick rate, measured in their ICLIF.

Your linguistic pedantry is wrong too, see elsewhere.
> > In principle (and nowadays almost in practice)
> > you can add a counter to each clock
> > that counts the individual wave crests directly.
> > (like on a Cesium clock)
> > Again, by just reading the digits you will see
> > that the higher clock runs faster,
> > without the need for making any connection between the clocks.
>
> For a given interval in the lab, you will see the upper clock
> accumulates more ticks than the lower clock. GR models this as due to
> their different paths through spacetime, not due to any difference in
> tick rates. This is DIFFERENT from the experiment discussed above (which
> involves tick rates).
>
> What you see with your eyes here is MIXING the ICLIFs of the two clocks,
> and you have ignored that, making your conclusion worthless.

That is just your unnecessary, and incorrect overhead.

> > Now every 'ordinary' person, and almost? all physicists,
> > will say that what we see here are two clocks running at slightly
> > different rates. (as predicted by general relativity)
>
> Your claim is WRONG. Physicists who understand GR will disagree with you
> and agree with what I said above. For the simple reason that I am
> describing how GR models this, and you are NOT.

People who really do this kind of thing strongly disagree with you.

> > Only you insist that the faster clock is not 'really' running faster.
>
> This is not "only me", it is how GR models these two different situations.
>
> GR is more complicated than you think, and much of what you think about
> it is WRONG. Your problem, not mine.

OK, so you have misunderstood what general relativity
has to say about it, and you remain in denial of observed reality.

No more to be said, I guess,

Jan

Re: If time goes slower for each twin

<1pkfppt.wgz7du11o5gq5N%nospam@de-ster.demon.nl>

  copy mid

https://www.novabbs.com/tech/article-flat.php?id=74901&group=sci.physics.relativity#74901

  copy link   Newsgroups: sci.physics.relativity
Path: i2pn2.org!i2pn.org!eternal-september.org!reader02.eternal-september.org!.POSTED!not-for-mail
From: nos...@de-ster.demon.nl (J. J. Lodder)
Newsgroups: sci.physics.relativity
Subject: Re: If time goes slower for each twin
Date: Sun, 19 Dec 2021 22:29:01 +0100
Organization: De Ster
Lines: 81
Message-ID: <1pkfppt.wgz7du11o5gq5N%nospam@de-ster.demon.nl>
References: <edb7909e-3fb6-40aa-b919-d164417c4383n@googlegroups.com> <sp7mlj$1kmk$1@gioia.aioe.org> <ef48c7ff-2589-4eba-8328-243a3402af47n@googlegroups.com> <1pk4xfm.9ayruk1i0tv8vN%nospam@de-ster.demon.nl> <8868015.CDJkKcVGEf@PointedEars.de> <1pk6cuj.1mz43ke4p1ilN%nospam@de-ster.demon.nl> <2223314.ElGaqSPkdT@PointedEars.de> <1pk8wy0.1hwtxny6rumisN%nospam@de-ster.demon.nl> <4489c1ea-05b3-43db-847a-a3644ce73c29n@googlegroups.com> <1pk94f3.1qgx6jp1ceyeocN%nospam@de-ster.demon.nl> <b246cde1-e36d-4448-8cad-e2d55a813018n@googlegroups.com> <1pk9z2x.1ncbcseon2cpbN%nospam@de-ster.demon.nl> <68bfdd09-e167-4e49-ba51-6b301e595190n@googlegroups.com> <1pk9zu9.14hdv7x1414yhoN%nospam@de-ster.demon.nl> <afOdnU7OHavWASb8nZ2dnUU7_83NnZ2d@giganews.com> <1pkauap.lnk9ul1adhmndN%nospam@de-ster.demon.nl> <OaudnSddyY_D5yD8nZ2dnUU7_8zNnZ2d@giganews.com> <1pkdq1a.1iq87uqnm3fmvN%nospam@de-ster.demon.nl> <1f449f0e-ad68-4de2-8f0b-f92d04463138n@googlegroups.com>
Reply-To: jjlax32@xs4all.nl (J. J. Lodder)
Injection-Info: reader02.eternal-september.org; posting-host="d6dfe58064eb127e906d52b2103683fc";
logging-data="23083"; mail-complaints-to="abuse@eternal-september.org"; posting-account="U2FsdGVkX19QINAmrcV/70Lzztt42eiBwWFiYXpy0XU="
User-Agent: MacSOUP/2.8.5 (ea919cf118) (Mac OS 10.10.5)
Cancel-Lock: sha1:dmT5V+kHQU18I64Mh/VAL4bNzMA=
 by: J. J. Lodder - Sun, 19 Dec 2021 21:29 UTC

Maciej Wozniak <maluwozniak@gmail.com> wrote:

> On Saturday, 18 December 2021 at 23:32:20 UTC+1, J. J. Lodder wrote:
> > Tom Roberts <tjrobe...@sbcglobal.net> wrote:
> >
> > > On 12/16/21 4:17 PM, J. J. Lodder wrote:
> > > > Tom Roberts <tjrobe...@sbcglobal.net> wrote:
> > > >> On 12/16/21 5:28 AM, J. J. Lodder wrote:
> > > >>> The second is nowadays defined as so many periods of .... -at
> > > >>> mean sea level-.
> > > >> This is WRONG. "-at mean sea level-" is NOT part of the
> > > >> definition. Go look it up -- YOU added that phrase. But that phrase
> > > >> CANNOT be included: A Cs-133 atomic clock will increment by one
> > > >> second for every 9,192,631,770 cycles of its ground-state
> > > >> hyperfine transition.
> > > >
> > > > But it is. You have failed to look up how TAI is defined.
> > >
> > > No. YOU have confused TAI with the definition of a second. Note that TAI
> > > uses the ISO definition of the second, and applies it on earth's geoid.
> > Now you are getting downright silly.
> > Do you really think that those peole at NIST have two different seconds,
> > a short second of their own and a longer TAI second?
> > > > And -the- second to be used for precision measurements is of course
> > > > the TAI second, not just any second from somewhere.
> > >
> > > No. That is just plain wrong.
> > >
> > > The second to be used for precision measurements is the second as
> > > defined by ISO. It may be convenient to derive it from TAI by applying
> > > the appropriate correction. (It's usually even more convenient to use
> > > GPS time; the same correction applies.)
> > ???? More sillyness. ISO has no time of its own,
> > it only deals with how to describe it.
> > > [...]
> > >
> > > >> this does NOT show that clocks in Paris tick more slowly than
> > > >> those in Boulder. Rather, it shows that the elapsed proper times of
> > > >> the two paths involved are different.
> > > >
> > > > Which is (with good reason) interpreted as time going slower in
> > > > Paris.
> > >
> > > In common speech, I suppose so. But not in a technical context like this.
> > No really. See my other posting.
> > Time does go slower in Paris than in Boulder.
> > Corrections are needed, and supplied by the BIPM
>
> Listen to your fellow idiot. You can't do these corrections;
> if you ask him "why? " he will derive it from the Holy Postulates.

It is precisely what NIST and BIPM and... have been doing,
for decades, by now.

> > Nope. They compare clocks directly.
> > Perhaps even clocks in the same room.
> > Latest feat: at Boulder they have succeeded in comparing the rates of
> > atoms in the same clock, with only 1mm height difference.
> > (taking frequency comparison to better than 10^-20)
> > Atoms at the top of the column tick faster than those at the bottom.
> > <https://arxiv.org/abs/2109.12238>
> > Clock design must take this fact into account.
>
> That's a horrible heresy, poor idiot.

It is what has been measured.

> > It escaped your notice that the rest frame of all clocks on Earth
> > is not an inertial coordinate frame.
> > But you are also wrong for clocks in orbit.
> > GPS clocks and Galileo clocks do need different corrections.
>
> Your idiot guru has postulated they don't, poor halfbrain.
> Do you deny his postulates?

No, I tell you what is observed.
And I tell you that this is precisely what GR predicts.

The only problem is that you don't understand it,

Jan

Re: If time goes slower for each twin

<spo8f4$efk$2@gioia.aioe.org>

  copy mid

https://www.novabbs.com/tech/article-flat.php?id=74903&group=sci.physics.relativity#74903

  copy link   Newsgroups: sci.physics.relativity
Path: i2pn2.org!i2pn.org!aioe.org!7a25jG6pUKCqa0zKnKnvdg.user.46.165.242.75.POSTED!not-for-mail
From: pyt...@example.invalid (Python)
Newsgroups: sci.physics.relativity
Subject: Re: If time goes slower for each twin
Date: Sun, 19 Dec 2021 22:33:24 +0100
Organization: Aioe.org NNTP Server
Message-ID: <spo8f4$efk$2@gioia.aioe.org>
References: <ade8707c-5779-44d2-9599-c42b4c2cdbf3n@googlegroups.com>
<1pk4fg7.6nqt0j3osf61N%nospam@de-ster.demon.nl>
<2088029.irdbgypaU6@PointedEars.de>
<1pk7zpw.1u4709u12ez725N%nospam@de-ster.demon.nl>
<jdadncq2e-PC5Cf8nZ2dnUU7_83NnZ2d@giganews.com>
<1pk9vhw.133tjv1sel3x2N%nospam@de-ster.demon.nl>
<ec11547f-e187-4a95-96f4-2bb1d4a309b4n@googlegroups.com>
<1pkac7z.1frr3nrtaz7ilN%nospam@de-ster.demon.nl>
<nK6dnY3H75YdDib8nZ2dnUU7_83NnZ2d@giganews.com>
<1pkell2.1h3cfauu373o3N%nospam@de-ster.demon.nl>
<t5udnfZSD4BtUCP8nZ2dnUU7_83NnZ2d@giganews.com>
<1pkfhvn.8y3k8armm3o0N%nospam@de-ster.demon.nl>
Mime-Version: 1.0
Content-Type: text/plain; charset=UTF-8; format=flowed
Content-Transfer-Encoding: 7bit
Injection-Info: gioia.aioe.org; logging-data="14836"; posting-host="7a25jG6pUKCqa0zKnKnvdg.user.gioia.aioe.org"; mail-complaints-to="abuse@aioe.org";
User-Agent: Mozilla/5.0 (Macintosh; Intel Mac OS X 10.13; rv:91.0)
Gecko/20100101 Thunderbird/91.4.0
X-Notice: Filtered by postfilter v. 0.9.2
Content-Language: fr
 by: Python - Sun, 19 Dec 2021 21:33 UTC

J. J. Lodder wrote:
> Tom Roberts <tjroberts137@sbcglobal.net> wrote:
>
>> On 12/18/21 4:32 PM, J. J. Lodder wrote:
>>> To make sure we talk about the same things I give a simple experiment.
>>> Suppose we have a lab, and several optical clocks.
>>> There is one on the lab bench, and suppose for convenience
>>> that it ticks at 500 THz precisely, so at 5*10^14 Hz.
>>> (with a stability of 10^-18)
>>> Now take a second identical clock,
>>> and place it on a pedestal about one meter higher on the bench.
>>> So with a \Delta\Phi of 10 m^2/s^2. (with c^2 ~= 10^17)
>>>
>>> By everybody's understanding of general relativity
>>> (and perhaps also by yours)
>>> it will tick at a rate of (1 + 10^-16) THz,
>>> so at 5 x 10^14 + 0.05 Hz.
>>
>> This is wrong. You are presuming your own MISUNDERSTANDING of General
>> Relativity. "Everybody" does not share your misunderstanding, and
>> physicists who understand GR will disagree with your claim. Each clock
>> will tick at 500 THz, because they are identical [#].
>
> So you are in denial of observed reality.
>
>> [#] I am talking about the clocks, AND ONLY THE CLOCKS.
>> So their tick rate is their intrinsic tick rate, which can
>> only be measured in their instantaneously co-movung locally
>> inertial frame (ICLIF). In this case, since the clocks are
>> so very accurate, 1 meter difference in altitude is enough
>> to make each one's ICLIF too small to include the other clock.
>
> The clocks are at rest with respect to each other,
> firmly bolted to the same lab table.
>
> But your whole ICLIF thing is a fundamental error.
> The difference in clock rates is NOT caused by the local gravity.
> It is not a local effect,
> so considering local inertial frames can't deal with it.
> If the geoid changes, so will the difference in clock rates between
> Boulder and Paris change.
>
>>> This difference in frequency is easily demonstrated.
>>> Take light from the upper clock,
>>> and bring it down with mirrors to the lower one,
>>> and let them interfere.
>>> You will see beats with a period of 20 seconds.
>>
>> Sure. But what is the effect of gravity on the light signal, and on the
>> way it is measured? You MUST include that in your analysis.
>
> Of course not. We just see the clocks ticking at different rates.
> Purely empirical, without any need for a theory.
>
>> GR is crystal clear: the beat is caused by the difference in the time
>> component of the metric between the clock and the observing instrument;
>> it appears in the geometrical projection of signal onto instrument. It
>> is NOT due to "identical clocks ticking at different rates" as you think.
>
> That's just your way of denying
> that you see clocks ticking at different rates,
> while standing next to them.
>
>>> Bring light from the lower one up, and idem.
>>> By direct frequency measurement using laser spectroscopy
>>> you can establish that the higher clock is faster,
>>> as predicted by general relativity.
>>
>> GR does not predict the upper clock is "faster", but it does predict the
>> beat, due to the way the interference is observed.
>
> Nice sophistery.
> They differ in rate because they beat, yet they run at the same rate.
> (but only in your mind's eye)
>
>> If you are going to apply "direct frequency measurement" to these
>> clocks, you need to connect each clock to a measuring instrument, and
>> must include the effect of gravitation on the signal between clock and
>> instrument.
>
> You clearly don't know how real life atomic clocks work.
> The counting mechanism is an inherent and integral part
> of the works inside.
>
>> If you do that by connecting each clock to an instrument
>> that is not moved, then the upper clock will be measured to tick faster
>> than the lower clock. If you then analyze the effect of gravity on those
>> signals, and correct the instrument's readings for it, you will find
>> both clocks actually tick at 500 THz [@].
>
> No signals are needed.
> Idealising: both clocks have running digits,
> counting periods of the clock,
> and you can see the higher one getting ahead,
> just by looking at them.
> (from whatever point you want to look)
>
>> [@] Again I remind you that this English phrase refers
>> just to the clocks, so it refers to their intrinsic
>> tick rate, measured in their ICLIF.
>
> Your linguistic pedantry is wrong too, see elsewhere.
>
>>> In principle (and nowadays almost in practice)
>>> you can add a counter to each clock
>>> that counts the individual wave crests directly.
>>> (like on a Cesium clock)
>>> Again, by just reading the digits you will see
>>> that the higher clock runs faster,
>>> without the need for making any connection between the clocks.
>>
>> For a given interval in the lab, you will see the upper clock
>> accumulates more ticks than the lower clock. GR models this as due to
>> their different paths through spacetime, not due to any difference in
>> tick rates. This is DIFFERENT from the experiment discussed above (which
>> involves tick rates).
>>
>> What you see with your eyes here is MIXING the ICLIFs of the two clocks,
>> and you have ignored that, making your conclusion worthless.
>
> That is just your unnecessary, and incorrect overhead.
>
>>> Now every 'ordinary' person, and almost? all physicists,
>>> will say that what we see here are two clocks running at slightly
>>> different rates. (as predicted by general relativity)
>>
>> Your claim is WRONG. Physicists who understand GR will disagree with you
>> and agree with what I said above. For the simple reason that I am
>> describing how GR models this, and you are NOT.
>
> People who really do this kind of thing strongly disagree with you.
>
>>> Only you insist that the faster clock is not 'really' running faster.
>>
>> This is not "only me", it is how GR models these two different situations.
>>
>> GR is more complicated than you think, and much of what you think about
>> it is WRONG. Your problem, not mine.
>
> OK, so you have misunderstood what general relativity
> has to say about it, and you remain in denial of observed reality.
>
> No more to be said, I guess,
>
> Jan

You should consider thinking about this, Jan. Tom is right.

Re: If time goes slower for each twin

<nuTYGqhdGzfxurXA2Cyb5GX3sR8@jntp>

  copy mid

https://www.novabbs.com/tech/article-flat.php?id=74906&group=sci.physics.relativity#74906

  copy link   Newsgroups: sci.physics.relativity
Path: i2pn2.org!i2pn.org!paganini.bofh.team!usenet.pasdenom.info!from-devjntp
Message-ID: <nuTYGqhdGzfxurXA2Cyb5GX3sR8@jntp>
JNTP-Route: news2.nemoweb.net
JNTP-DataType: Article
Subject: Re: If time goes slower for each twin
References: <ade8707c-5779-44d2-9599-c42b4c2cdbf3n@googlegroups.com> <jdadncq2e-PC5Cf8nZ2dnUU7_83NnZ2d@giganews.com>
<1pk9vhw.133tjv1sel3x2N%nospam@de-ster.demon.nl> <ec11547f-e187-4a95-96f4-2bb1d4a309b4n@googlegroups.com>
<1pkac7z.1frr3nrtaz7ilN%nospam@de-ster.demon.nl> <nK6dnY3H75YdDib8nZ2dnUU7_83NnZ2d@giganews.com>
<1pkell2.1h3cfauu373o3N%nospam@de-ster.demon.nl> <t5udnfZSD4BtUCP8nZ2dnUU7_83NnZ2d@giganews.com>
<1pkfhvn.8y3k8armm3o0N%nospam@de-ster.demon.nl> <spo8f4$efk$2@gioia.aioe.org>
Newsgroups: sci.physics.relativity
JNTP-HashClient: YShKUIZZq1f5EvJRyqVDbAf8_Kg
JNTP-ThreadID: ade8707c-5779-44d2-9599-c42b4c2cdbf3n@googlegroups.com
JNTP-Uri: http://news2.nemoweb.net/?DataID=nuTYGqhdGzfxurXA2Cyb5GX3sR8@jntp
User-Agent: Nemo/0.999a
JNTP-OriginServer: news2.nemoweb.net
Date: Sun, 19 Dec 21 21:41:40 +0000
Organization: Nemoweb
JNTP-Browser: Mozilla/5.0 (Windows NT 10.0; Win64; x64) AppleWebKit/537.36 (KHTML, like Gecko) Chrome/96.0.4664.110 Safari/537.36
Injection-Info: news2.nemoweb.net; posting-host="16a01200b07d7bf03c4129f998bc72c217140961"; logging-data="2021-12-19T21:41:40Z/6404215"; posting-account="4@news2.nemoweb.net"; mail-complaints-to="newsmaster@news2.nemoweb.net"
JNTP-ProtocolVersion: 0.21.1
JNTP-Server: PhpNemoServer/0.94.5
MIME-Version: 1.0
Content-Type: text/plain; charset=UTF-8; format=flowed
Content-Transfer-Encoding: 8bit
X-JNTP-JsonNewsGateway: 0.96
From: r.hac...@tiscali.fr (Richard Hachel)
 by: Richard Hachel - Sun, 19 Dec 2021 21:41 UTC

Le 19/12/2021 à 22:33, Jean-Pierre Messager as Python a écrit :

> Tom is right.

C'est là que je commence à avoir des doutes sur Tom. :))

R.H.

Re: If time goes slower for each twin

<1pkgi64.jiwfk5rc4o01N%nospam@de-ster.demon.nl>

  copy mid

https://www.novabbs.com/tech/article-flat.php?id=74910&group=sci.physics.relativity#74910

  copy link   Newsgroups: sci.physics.relativity
Path: i2pn2.org!i2pn.org!eternal-september.org!reader02.eternal-september.org!.POSTED!not-for-mail
From: nos...@de-ster.demon.nl (J. J. Lodder)
Newsgroups: sci.physics.relativity
Subject: Re: If time goes slower for each twin
Date: Sun, 19 Dec 2021 22:58:09 +0100
Organization: De Ster
Lines: 163
Message-ID: <1pkgi64.jiwfk5rc4o01N%nospam@de-ster.demon.nl>
References: <ade8707c-5779-44d2-9599-c42b4c2cdbf3n@googlegroups.com> <1pk4fg7.6nqt0j3osf61N%nospam@de-ster.demon.nl> <2088029.irdbgypaU6@PointedEars.de> <1pk7zpw.1u4709u12ez725N%nospam@de-ster.demon.nl> <jdadncq2e-PC5Cf8nZ2dnUU7_83NnZ2d@giganews.com> <1pk9vhw.133tjv1sel3x2N%nospam@de-ster.demon.nl> <ec11547f-e187-4a95-96f4-2bb1d4a309b4n@googlegroups.com> <1pkac7z.1frr3nrtaz7ilN%nospam@de-ster.demon.nl> <nK6dnY3H75YdDib8nZ2dnUU7_83NnZ2d@giganews.com> <1pkell2.1h3cfauu373o3N%nospam@de-ster.demon.nl> <t5udnfZSD4BtUCP8nZ2dnUU7_83NnZ2d@giganews.com> <1pkfhvn.8y3k8armm3o0N%nospam@de-ster.demon.nl> <spo8f4$efk$2@gioia.aioe.org>
Reply-To: jjlax32@xs4all.nl (J. J. Lodder)
Injection-Info: reader02.eternal-september.org; posting-host="d6dfe58064eb127e906d52b2103683fc";
logging-data="8258"; mail-complaints-to="abuse@eternal-september.org"; posting-account="U2FsdGVkX18QElsqBUbzeOqt92IbyRddtxFuSOh8GYE="
User-Agent: MacSOUP/2.8.5 (ea919cf118) (Mac OS 10.10.5)
Cancel-Lock: sha1:g5Gx1BrRkYcXJ7VAXEDZN068wsY=
 by: J. J. Lodder - Sun, 19 Dec 2021 21:58 UTC

Python <python@example.invalid> wrote:

> J. J. Lodder wrote:
> > Tom Roberts <tjroberts137@sbcglobal.net> wrote:
> >
> >> On 12/18/21 4:32 PM, J. J. Lodder wrote:
> >>> To make sure we talk about the same things I give a simple experiment.
> >>> Suppose we have a lab, and several optical clocks.
> >>> There is one on the lab bench, and suppose for convenience
> >>> that it ticks at 500 THz precisely, so at 5*10^14 Hz.
> >>> (with a stability of 10^-18)
> >>> Now take a second identical clock,
> >>> and place it on a pedestal about one meter higher on the bench.
> >>> So with a \Delta\Phi of 10 m^2/s^2. (with c^2 ~= 10^17)
> >>>
> >>> By everybody's understanding of general relativity
> >>> (and perhaps also by yours)
> >>> it will tick at a rate of (1 + 10^-16) THz,
> >>> so at 5 x 10^14 + 0.05 Hz.
> >>
> >> This is wrong. You are presuming your own MISUNDERSTANDING of General
> >> Relativity. "Everybody" does not share your misunderstanding, and
> >> physicists who understand GR will disagree with your claim. Each clock
> >> will tick at 500 THz, because they are identical [#].
> >
> > So you are in denial of observed reality.
> >
> >> [#] I am talking about the clocks, AND ONLY THE CLOCKS.
> >> So their tick rate is their intrinsic tick rate, which can
> >> only be measured in their instantaneously co-movung locally
> >> inertial frame (ICLIF). In this case, since the clocks are
> >> so very accurate, 1 meter difference in altitude is enough
> >> to make each one's ICLIF too small to include the other clock.
> >
> > The clocks are at rest with respect to each other,
> > firmly bolted to the same lab table.
> >
> > But your whole ICLIF thing is a fundamental error.
> > The difference in clock rates is NOT caused by the local gravity.
> > It is not a local effect,
> > so considering local inertial frames can't deal with it.
> > If the geoid changes, so will the difference in clock rates between
> > Boulder and Paris change.
> >
> >>> This difference in frequency is easily demonstrated.
> >>> Take light from the upper clock,
> >>> and bring it down with mirrors to the lower one,
> >>> and let them interfere.
> >>> You will see beats with a period of 20 seconds.
> >>
> >> Sure. But what is the effect of gravity on the light signal, and on the
> >> way it is measured? You MUST include that in your analysis.
> >
> > Of course not. We just see the clocks ticking at different rates.
> > Purely empirical, without any need for a theory.
> >
> >> GR is crystal clear: the beat is caused by the difference in the time
> >> component of the metric between the clock and the observing instrument;
> >> it appears in the geometrical projection of signal onto instrument. It
> >> is NOT due to "identical clocks ticking at different rates" as you think.
> >
> > That's just your way of denying
> > that you see clocks ticking at different rates,
> > while standing next to them.
> >
> >>> Bring light from the lower one up, and idem.
> >>> By direct frequency measurement using laser spectroscopy
> >>> you can establish that the higher clock is faster,
> >>> as predicted by general relativity.
> >>
> >> GR does not predict the upper clock is "faster", but it does predict the
> >> beat, due to the way the interference is observed.
> >
> > Nice sophistery.
> > They differ in rate because they beat, yet they run at the same rate.
> > (but only in your mind's eye)
> >
> >> If you are going to apply "direct frequency measurement" to these
> >> clocks, you need to connect each clock to a measuring instrument, and
> >> must include the effect of gravitation on the signal between clock and
> >> instrument.
> >
> > You clearly don't know how real life atomic clocks work.
> > The counting mechanism is an inherent and integral part
> > of the works inside.
> >
> >> If you do that by connecting each clock to an instrument
> >> that is not moved, then the upper clock will be measured to tick faster
> >> than the lower clock. If you then analyze the effect of gravity on those
> >> signals, and correct the instrument's readings for it, you will find
> >> both clocks actually tick at 500 THz [@].
> >
> > No signals are needed.
> > Idealising: both clocks have running digits,
> > counting periods of the clock,
> > and you can see the higher one getting ahead,
> > just by looking at them.
> > (from whatever point you want to look)
> >
> >> [@] Again I remind you that this English phrase refers
> >> just to the clocks, so it refers to their intrinsic
> >> tick rate, measured in their ICLIF.
> >
> > Your linguistic pedantry is wrong too, see elsewhere.
> >
> >>> In principle (and nowadays almost in practice)
> >>> you can add a counter to each clock
> >>> that counts the individual wave crests directly.
> >>> (like on a Cesium clock)
> >>> Again, by just reading the digits you will see
> >>> that the higher clock runs faster,
> >>> without the need for making any connection between the clocks.
> >>
> >> For a given interval in the lab, you will see the upper clock
> >> accumulates more ticks than the lower clock. GR models this as due to
> >> their different paths through spacetime, not due to any difference in
> >> tick rates. This is DIFFERENT from the experiment discussed above (which
> >> involves tick rates).
> >>
> >> What you see with your eyes here is MIXING the ICLIFs of the two clocks,
> >> and you have ignored that, making your conclusion worthless.
> >
> > That is just your unnecessary, and incorrect overhead.
> >
> >>> Now every 'ordinary' person, and almost? all physicists,
> >>> will say that what we see here are two clocks running at slightly
> >>> different rates. (as predicted by general relativity)
> >>
> >> Your claim is WRONG. Physicists who understand GR will disagree with you
> >> and agree with what I said above. For the simple reason that I am
> >> describing how GR models this, and you are NOT.
> >
> > People who really do this kind of thing strongly disagree with you.
> >
> >>> Only you insist that the faster clock is not 'really' running faster.
> >>
> >> This is not "only me", it is how GR models these two different situations.
> >>
> >> GR is more complicated than you think, and much of what you think about
> >> it is WRONG. Your problem, not mine.
> >
> > OK, so you have misunderstood what general relativity
> > has to say about it, and you remain in denial of observed reality.
> >
> > No more to be said, I guess,
> >
> > Jan
>
>
> You should consider thinking about this, Jan. Tom is right.

Nope. Clocks at different altitudes do tick at different rates,
and anyone who has the right tools can measure that.
You might want to search on 'relativistic geodesy',
or 'chronometric levelling' for some of the facts.

We will soon be able to map out the geoid
to a new level of accuracy,
by measuring differences in clock rates,
all over the planet,

Jan

Re: If time goes slower for each twin

<spoal7$1dmg$1@gioia.aioe.org>

  copy mid

https://www.novabbs.com/tech/article-flat.php?id=74912&group=sci.physics.relativity#74912

  copy link   Newsgroups: sci.physics.relativity
Path: i2pn2.org!i2pn.org!aioe.org!7a25jG6pUKCqa0zKnKnvdg.user.46.165.242.75.POSTED!not-for-mail
From: pyt...@example.invalid (Python)
Newsgroups: sci.physics.relativity
Subject: Re: If time goes slower for each twin
Date: Sun, 19 Dec 2021 23:10:47 +0100
Organization: Aioe.org NNTP Server
Message-ID: <spoal7$1dmg$1@gioia.aioe.org>
References: <ade8707c-5779-44d2-9599-c42b4c2cdbf3n@googlegroups.com>
<1pk4fg7.6nqt0j3osf61N%nospam@de-ster.demon.nl>
<2088029.irdbgypaU6@PointedEars.de>
<1pk7zpw.1u4709u12ez725N%nospam@de-ster.demon.nl>
<jdadncq2e-PC5Cf8nZ2dnUU7_83NnZ2d@giganews.com>
<1pk9vhw.133tjv1sel3x2N%nospam@de-ster.demon.nl>
<ec11547f-e187-4a95-96f4-2bb1d4a309b4n@googlegroups.com>
<1pkac7z.1frr3nrtaz7ilN%nospam@de-ster.demon.nl>
<nK6dnY3H75YdDib8nZ2dnUU7_83NnZ2d@giganews.com>
<1pkell2.1h3cfauu373o3N%nospam@de-ster.demon.nl>
<t5udnfZSD4BtUCP8nZ2dnUU7_83NnZ2d@giganews.com>
<1pkfhvn.8y3k8armm3o0N%nospam@de-ster.demon.nl> <spo8f4$efk$2@gioia.aioe.org>
<1pkgi64.jiwfk5rc4o01N%nospam@de-ster.demon.nl>
Mime-Version: 1.0
Content-Type: text/plain; charset=UTF-8; format=flowed
Content-Transfer-Encoding: 7bit
Injection-Info: gioia.aioe.org; logging-data="46800"; posting-host="7a25jG6pUKCqa0zKnKnvdg.user.gioia.aioe.org"; mail-complaints-to="abuse@aioe.org";
User-Agent: Mozilla/5.0 (Macintosh; Intel Mac OS X 10.13; rv:91.0)
Gecko/20100101 Thunderbird/91.4.0
Content-Language: fr
X-Notice: Filtered by postfilter v. 0.9.2
 by: Python - Sun, 19 Dec 2021 22:10 UTC

J. J. Lodder wrote:
> Python <python@example.invalid> wrote:
>
>> J. J. Lodder wrote:
>>> Tom Roberts <tjroberts137@sbcglobal.net> wrote:
>>>
>>>> On 12/18/21 4:32 PM, J. J. Lodder wrote:
>>>>> To make sure we talk about the same things I give a simple experiment.
>>>>> Suppose we have a lab, and several optical clocks.
>>>>> There is one on the lab bench, and suppose for convenience
>>>>> that it ticks at 500 THz precisely, so at 5*10^14 Hz.
>>>>> (with a stability of 10^-18)
>>>>> Now take a second identical clock,
>>>>> and place it on a pedestal about one meter higher on the bench.
>>>>> So with a \Delta\Phi of 10 m^2/s^2. (with c^2 ~= 10^17)
>>>>>
>>>>> By everybody's understanding of general relativity
>>>>> (and perhaps also by yours)
>>>>> it will tick at a rate of (1 + 10^-16) THz,
>>>>> so at 5 x 10^14 + 0.05 Hz.
>>>>
>>>> This is wrong. You are presuming your own MISUNDERSTANDING of General
>>>> Relativity. "Everybody" does not share your misunderstanding, and
>>>> physicists who understand GR will disagree with your claim. Each clock
>>>> will tick at 500 THz, because they are identical [#].
>>>
>>> So you are in denial of observed reality.
>>>
>>>> [#] I am talking about the clocks, AND ONLY THE CLOCKS.
>>>> So their tick rate is their intrinsic tick rate, which can
>>>> only be measured in their instantaneously co-movung locally
>>>> inertial frame (ICLIF). In this case, since the clocks are
>>>> so very accurate, 1 meter difference in altitude is enough
>>>> to make each one's ICLIF too small to include the other clock.
>>>
>>> The clocks are at rest with respect to each other,
>>> firmly bolted to the same lab table.
>>>
>>> But your whole ICLIF thing is a fundamental error.
>>> The difference in clock rates is NOT caused by the local gravity.
>>> It is not a local effect,
>>> so considering local inertial frames can't deal with it.
>>> If the geoid changes, so will the difference in clock rates between
>>> Boulder and Paris change.
>>>
>>>>> This difference in frequency is easily demonstrated.
>>>>> Take light from the upper clock,
>>>>> and bring it down with mirrors to the lower one,
>>>>> and let them interfere.
>>>>> You will see beats with a period of 20 seconds.
>>>>
>>>> Sure. But what is the effect of gravity on the light signal, and on the
>>>> way it is measured? You MUST include that in your analysis.
>>>
>>> Of course not. We just see the clocks ticking at different rates.
>>> Purely empirical, without any need for a theory.
>>>
>>>> GR is crystal clear: the beat is caused by the difference in the time
>>>> component of the metric between the clock and the observing instrument;
>>>> it appears in the geometrical projection of signal onto instrument. It
>>>> is NOT due to "identical clocks ticking at different rates" as you think.
>>>
>>> That's just your way of denying
>>> that you see clocks ticking at different rates,
>>> while standing next to them.
>>>
>>>>> Bring light from the lower one up, and idem.
>>>>> By direct frequency measurement using laser spectroscopy
>>>>> you can establish that the higher clock is faster,
>>>>> as predicted by general relativity.
>>>>
>>>> GR does not predict the upper clock is "faster", but it does predict the
>>>> beat, due to the way the interference is observed.
>>>
>>> Nice sophistery.
>>> They differ in rate because they beat, yet they run at the same rate.
>>> (but only in your mind's eye)
>>>
>>>> If you are going to apply "direct frequency measurement" to these
>>>> clocks, you need to connect each clock to a measuring instrument, and
>>>> must include the effect of gravitation on the signal between clock and
>>>> instrument.
>>>
>>> You clearly don't know how real life atomic clocks work.
>>> The counting mechanism is an inherent and integral part
>>> of the works inside.
>>>
>>>> If you do that by connecting each clock to an instrument
>>>> that is not moved, then the upper clock will be measured to tick faster
>>>> than the lower clock. If you then analyze the effect of gravity on those
>>>> signals, and correct the instrument's readings for it, you will find
>>>> both clocks actually tick at 500 THz [@].
>>>
>>> No signals are needed.
>>> Idealising: both clocks have running digits,
>>> counting periods of the clock,
>>> and you can see the higher one getting ahead,
>>> just by looking at them.
>>> (from whatever point you want to look)
>>>
>>>> [@] Again I remind you that this English phrase refers
>>>> just to the clocks, so it refers to their intrinsic
>>>> tick rate, measured in their ICLIF.
>>>
>>> Your linguistic pedantry is wrong too, see elsewhere.
>>>
>>>>> In principle (and nowadays almost in practice)
>>>>> you can add a counter to each clock
>>>>> that counts the individual wave crests directly.
>>>>> (like on a Cesium clock)
>>>>> Again, by just reading the digits you will see
>>>>> that the higher clock runs faster,
>>>>> without the need for making any connection between the clocks.
>>>>
>>>> For a given interval in the lab, you will see the upper clock
>>>> accumulates more ticks than the lower clock. GR models this as due to
>>>> their different paths through spacetime, not due to any difference in
>>>> tick rates. This is DIFFERENT from the experiment discussed above (which
>>>> involves tick rates).
>>>>
>>>> What you see with your eyes here is MIXING the ICLIFs of the two clocks,
>>>> and you have ignored that, making your conclusion worthless.
>>>
>>> That is just your unnecessary, and incorrect overhead.
>>>
>>>>> Now every 'ordinary' person, and almost? all physicists,
>>>>> will say that what we see here are two clocks running at slightly
>>>>> different rates. (as predicted by general relativity)
>>>>
>>>> Your claim is WRONG. Physicists who understand GR will disagree with you
>>>> and agree with what I said above. For the simple reason that I am
>>>> describing how GR models this, and you are NOT.
>>>
>>> People who really do this kind of thing strongly disagree with you.
>>>
>>>>> Only you insist that the faster clock is not 'really' running faster.
>>>>
>>>> This is not "only me", it is how GR models these two different situations.
>>>>
>>>> GR is more complicated than you think, and much of what you think about
>>>> it is WRONG. Your problem, not mine.
>>>
>>> OK, so you have misunderstood what general relativity
>>> has to say about it, and you remain in denial of observed reality.
>>>
>>> No more to be said, I guess,
>>>
>>> Jan
>>
>>
>> You should consider thinking about this, Jan. Tom is right.
>
> Nope. Clocks at different altitudes do tick at different rates,
> and anyone who has the right tools can measure that.
> You might want to search on 'relativistic geodesy',
> or 'chronometric levelling' for some of the facts.
>
> We will soon be able to map out the geoid
> to a new level of accuracy,
> by measuring differences in clock rates,
> all over the planet,
>
> Jan

Jan, think about the meaning of words.

Re: If time goes slower for each twin

<36e4e94f-48e6-43a0-baae-e8f3e3865478n@googlegroups.com>

  copy mid

https://www.novabbs.com/tech/article-flat.php?id=74925&group=sci.physics.relativity#74925

  copy link   Newsgroups: sci.physics.relativity
X-Received: by 2002:a05:622a:1654:: with SMTP id y20mr354460qtj.374.1639961394776;
Sun, 19 Dec 2021 16:49:54 -0800 (PST)
X-Received: by 2002:a05:620a:4687:: with SMTP id bq7mr4105406qkb.540.1639961394543;
Sun, 19 Dec 2021 16:49:54 -0800 (PST)
Path: i2pn2.org!i2pn.org!weretis.net!feeder6.news.weretis.net!news.misty.com!border2.nntp.dca1.giganews.com!nntp.giganews.com!news-out.google.com!nntp.google.com!postnews.google.com!google-groups.googlegroups.com!not-for-mail
Newsgroups: sci.physics.relativity
Date: Sun, 19 Dec 2021 16:49:54 -0800 (PST)
In-Reply-To: <1pkfhvn.8y3k8armm3o0N%nospam@de-ster.demon.nl>
Injection-Info: google-groups.googlegroups.com; posting-host=205.154.192.197; posting-account=x2WXVAkAAACheXC-5ndnEdz_vL9CA75q
NNTP-Posting-Host: 205.154.192.197
References: <ade8707c-5779-44d2-9599-c42b4c2cdbf3n@googlegroups.com>
<1pk4fg7.6nqt0j3osf61N%nospam@de-ster.demon.nl> <2088029.irdbgypaU6@PointedEars.de>
<1pk7zpw.1u4709u12ez725N%nospam@de-ster.demon.nl> <jdadncq2e-PC5Cf8nZ2dnUU7_83NnZ2d@giganews.com>
<1pk9vhw.133tjv1sel3x2N%nospam@de-ster.demon.nl> <ec11547f-e187-4a95-96f4-2bb1d4a309b4n@googlegroups.com>
<1pkac7z.1frr3nrtaz7ilN%nospam@de-ster.demon.nl> <nK6dnY3H75YdDib8nZ2dnUU7_83NnZ2d@giganews.com>
<1pkell2.1h3cfauu373o3N%nospam@de-ster.demon.nl> <t5udnfZSD4BtUCP8nZ2dnUU7_83NnZ2d@giganews.com>
<1pkfhvn.8y3k8armm3o0N%nospam@de-ster.demon.nl>
User-Agent: G2/1.0
MIME-Version: 1.0
Message-ID: <36e4e94f-48e6-43a0-baae-e8f3e3865478n@googlegroups.com>
Subject: Re: If time goes slower for each twin
From: r_delane...@yahoo.com (RichD)
Injection-Date: Mon, 20 Dec 2021 00:49:54 +0000
Content-Type: text/plain; charset="UTF-8"
Lines: 40
 by: RichD - Mon, 20 Dec 2021 00:49 UTC

On December 19, J. J. Lodder wrote:
>>> Suppose we have a lab, and several optical clocks.
>>> There is one on the lab bench, and suppose
>>> that it ticks at 500 THz precisely, so at 5*10^14 Hz
>>> Now take a second identical clock,
>>> and place it on a pedestal about one meter higher on the bench.
>>> So with a \Delta\Phi of 10 m^2/s^2
>>> By everybody's understanding of general relativity
>>> it will tick at a rate of (1 + 10^-16) THz,
>>> so at 5 x 10^14 + 0.05 Hz.
>
>> [#] I am talking about the clocks, AND ONLY THE CLOCKS.
>> So their tick rate is their intrinsic tick rate, which can
>> only be measured in their instantaneously co-movung locally
>> inertial frame (ICLIF). In this case, since the clocks are
>> so very accurate, 1 meter difference in altitude is enough
>> to make each one's ICLIF too small to include the other clock.
>
> The clocks are at rest with respect to each other,
> firmly bolted to the same lab table.
> But your whole ICLIF thing is a fundamental error.
> The difference in clock rates is NOT caused by the local gravity.

Now let me get this straight:
One clock is at a higher altitude, by one meter.
The higher altitude unit ticks faster, as observed in the lab.
General relativity predicts this, by "everybody's understanding".
But it's NOT caused by a difference in local gravity.

"You humans are quite illogical."
- Mr. Spock

> OK, so you have misunderstood what general relativity
> has to say about it, and you remain in denial of observed reality.
> No more to be said, I guess,

I say it's leprechauns - if it isn't due to gravity, leprechauns are
the only thing left -

--
Rich

Re: If time goes slower for each twin

<j2amplF4a1uU1@mid.individual.net>

  copy mid

https://www.novabbs.com/tech/article-flat.php?id=74937&group=sci.physics.relativity#74937

  copy link   Newsgroups: sci.physics.relativity
Path: i2pn2.org!i2pn.org!weretis.net!feeder8.news.weretis.net!3.eu.feeder.erje.net!feeder.erje.net!fu-berlin.de!uni-berlin.de!individual.net!not-for-mail
From: ttt_...@web.de (Thomas Heger)
Newsgroups: sci.physics.relativity
Subject: Re: If time goes slower for each twin
Date: Mon, 20 Dec 2021 07:39:52 +0100
Lines: 72
Message-ID: <j2amplF4a1uU1@mid.individual.net>
References: <ade8707c-5779-44d2-9599-c42b4c2cdbf3n@googlegroups.com> <sp4mus$78a$1@gioia.aioe.org> <O90D20WUu984acPU1w_S90qqcuU@jntp> <4676671.GXAFRqVoOG@PointedEars.de> <6MmCLEceSpwh8yfl41Jlt8j_tlA@jntp> <UXW1-e65QWxhFcysGp_wEQcIDvM@jntp> <2089212.irdbgypaU6@PointedEars.de> <EQ3l7crtJC_TXLNyfniK_mjBZSo@jntp> <12914471.uLZWGnKmhe@PointedEars.de> <RHSgLwi_4mlmyKU2Qx7KMAUkVkY@jntp> <03e8378e-c4e4-469b-9316-15e3d9d5aab1n@googlegroups.com> <v-Z96JM7oeo3z2tAH3OacbwhD2E@jntp> <sp7jnu$200$3@gioia.aioe.org> <edb7909e-3fb6-40aa-b919-d164417c4383n@googlegroups.com> <sp7mlj$1kmk$1@gioia.aioe.org> <ef48c7ff-2589-4eba-8328-243a3402af47n@googlegroups.com> <1pk4xfm.9ayruk1i0tv8vN%nospam@de-ster.demon.nl> <8868015.CDJkKcVGEf@PointedEars.de> <1pk6cuj.1mz43ke4p1ilN%nospam@de-ster.demon.nl> <2223314.ElGaqSPkdT@PointedEars.de> <1pk8wy0.1hwtxny6rumisN%nospam@de-ster.demon.nl> <4c77b5bc-722f-4fc7-aae6-43e7663b1569n@googlegroups.com> <j29gloFs1gmU1@mid.individual.net> <j29iglFsd1hU1@mid.individual.net>
Mime-Version: 1.0
Content-Type: text/plain; charset=utf-8; format=flowed
Content-Transfer-Encoding: 7bit
X-Trace: individual.net 1m6X+keO05zCbgkWdYDisAqBNt/UiaZCXbeaE3kHYm1HqkMQHk
Cancel-Lock: sha1:nO6mR445IG4cFGm6wwwIGlUwMNE=
User-Agent: Mozilla/5.0 (Windows NT 6.0; WOW64; rv:31.0) Gecko/20100101 Thunderbird/31.4.0
In-Reply-To: <j29iglFsd1hU1@mid.individual.net>
 by: Thomas Heger - Mon, 20 Dec 2021 06:39 UTC

Am 19.12.2021 um 21:20 schrieb whodat:
> On 12/19/2021 1:49 PM, Thomas Heger wrote:
>> Am 19.12.2021 um 02:44 schrieb RichD:
>>> On December 15, J. J. Lodder wrote:
>>>>>> Certainly, each clock runs at its own rate in its own local rest
>>>>>> frame.
>>>
>>> Define "runs at its own rate"
>>
>>
>> We should take time as a local measure and reject the idea of
>> universal time.
>>
>> We know, that here on Earth the clocks run all into the same direction
>> (into our local future) and tick with the same rate.
>>
>>>>> But the reason is NOT that one clock is ticking faster than the
>>>>> other, but
>>>>> that one clock measures *more elapsed proper time* than the other.
>>>>
>>>> There is nothing relative about it: clocks that stand higher run
>>>> faster,
>>>> objectively, absolutely, and for all observers.
>>>
>>> Define "runs faster"
>>
>>
>> THIS is a difficult problem:
>>
>> I matter is 'relative' we have also 'relative time'.
>>
>> These different time-axes define different 'time domaines'.
>>
>> since absolute clocks do not exist, the local clocks can only be
>> compared to other clocks, which eventually tick at a different rate.
>>
>> But in this case we can only say, which clock is faster than the
>> other. But we cannot decide, wether our clocks run fast (or not).
>
> You can't have it both ways. First you say that all earth clocks tick at
> the same rate, then you question the ticking rate of clocks saying
> that other clocks eventually tick at a different rate without
> establishing their conditions.

Other clocks from other universes may have any conceivable tick rate.

We could only compare the tick-rate from there to our clocks and decide
upon that comparison, if time in such a universe would run faster or
slower (compared to ours).

But we cannot compare our own tick-rate of our clocks with anything, if
we have nothing to compare with.

iow: we cannot say, whether our local time is fast or slow, if we have
no universe to compare our clocks with.

>
> Hint: When creating a posting, make sure it is consistent within itself.
> Then make sure all the implications are consistent.
>
> I've only recently come to read this group, and much of what I have seen
> here isn't worth the time. I need a better hobby.

I think, that theoretical physics is a good hobby, because it keeps the
brain in training.

But it requires some degree of masochism, because good results will gain
nothing but trouble and tons of insults.

TH

Re: If time goes slower for each twin

<7d790846-2526-40dd-abe8-551ef6faa27dn@googlegroups.com>

  copy mid

https://www.novabbs.com/tech/article-flat.php?id=74939&group=sci.physics.relativity#74939

  copy link   Newsgroups: sci.physics.relativity
X-Received: by 2002:a05:620a:b8b:: with SMTP id k11mr8537478qkh.746.1639982884191;
Sun, 19 Dec 2021 22:48:04 -0800 (PST)
X-Received: by 2002:a05:620a:2894:: with SMTP id j20mr8850335qkp.307.1639982884046;
Sun, 19 Dec 2021 22:48:04 -0800 (PST)
Path: i2pn2.org!i2pn.org!weretis.net!feeder6.news.weretis.net!news.misty.com!border2.nntp.dca1.giganews.com!nntp.giganews.com!news-out.google.com!nntp.google.com!postnews.google.com!google-groups.googlegroups.com!not-for-mail
Newsgroups: sci.physics.relativity
Date: Sun, 19 Dec 2021 22:48:03 -0800 (PST)
In-Reply-To: <1pkfhrc.aw7su3eyz58rN%nospam@de-ster.demon.nl>
Injection-Info: google-groups.googlegroups.com; posting-host=89.206.14.16; posting-account=I3DWzAoAAACOmZUdDcZ-C0PqAZGVsbW0
NNTP-Posting-Host: 89.206.14.16
References: <edb7909e-3fb6-40aa-b919-d164417c4383n@googlegroups.com>
<sp7mlj$1kmk$1@gioia.aioe.org> <ef48c7ff-2589-4eba-8328-243a3402af47n@googlegroups.com>
<1pk4xfm.9ayruk1i0tv8vN%nospam@de-ster.demon.nl> <8868015.CDJkKcVGEf@PointedEars.de>
<1pk6cuj.1mz43ke4p1ilN%nospam@de-ster.demon.nl> <2223314.ElGaqSPkdT@PointedEars.de>
<1pk8wy0.1hwtxny6rumisN%nospam@de-ster.demon.nl> <4489c1ea-05b3-43db-847a-a3644ce73c29n@googlegroups.com>
<1pk94f3.1qgx6jp1ceyeocN%nospam@de-ster.demon.nl> <b246cde1-e36d-4448-8cad-e2d55a813018n@googlegroups.com>
<1pk9z2x.1ncbcseon2cpbN%nospam@de-ster.demon.nl> <68bfdd09-e167-4e49-ba51-6b301e595190n@googlegroups.com>
<1pk9zu9.14hdv7x1414yhoN%nospam@de-ster.demon.nl> <afOdnU7OHavWASb8nZ2dnUU7_83NnZ2d@giganews.com>
<1pkauap.lnk9ul1adhmndN%nospam@de-ster.demon.nl> <OaudnSddyY_D5yD8nZ2dnUU7_8zNnZ2d@giganews.com>
<1pkdq1a.1iq87uqnm3fmvN%nospam@de-ster.demon.nl> <spm5mi$363$1@gioia.aioe.org>
<TeSdncTdEok_JSP8nZ2dnUU7_83NnZ2d@giganews.com> <1pkfhrc.aw7su3eyz58rN%nospam@de-ster.demon.nl>
User-Agent: G2/1.0
MIME-Version: 1.0
Message-ID: <7d790846-2526-40dd-abe8-551ef6faa27dn@googlegroups.com>
Subject: Re: If time goes slower for each twin
From: maluwozn...@gmail.com (Maciej Wozniak)
Injection-Date: Mon, 20 Dec 2021 06:48:04 +0000
Content-Type: text/plain; charset="UTF-8"
Lines: 32
 by: Maciej Wozniak - Mon, 20 Dec 2021 06:48 UTC

On Sunday, 19 December 2021 at 22:29:01 UTC+1, J. J. Lodder wrote:
> Tom Roberts <tjrobe...@sbcglobal.net> wrote:
>
> > On 12/18/21 8:33 PM, Michael Moroney wrote:
> > > clocks tick at their standard rate. It is only comparing signals
> > > from other clocks under different circumstances where this could be
> > > stated. Someone in Boulder would say the clock in Paris runs slow
> > > and they'd be correct, for Boulder. Someone in Paris would say the
> > > Boulder clock runs fast, and *they'd* be correct, for Paris (or
> > > London etc.)
> >
> > No. You used incorrect English and missed the essential point. A person
> > in Boulder would say the clock in Paris runs slow COMPARED TO MY CLOCK.
> > A person in Paris would say the Boulder clock runs fast COMPARED TO MY
> > CLOCK. And if used for scientific purposes they would describe how the
> > comparisons are made.
> >
> > Basic English: "slow" and "fast" are comparisons [#], and here they
> > necessarily relate two clocks to each other.
> >
> > [#] More properly, "slower" and "faster", but both forms
> > are used.
> Now you have nothing better left than carping about use of English?
>
> 'Slow' and 'fast' do not need a 'compared to' when there is a well
> understood common standard.
> For atomic clocks this is of course TAI.

Of course not. TAI violates the Laws of the Nature,
announced by your idiot guru!!!
He has postulated clocks to be identical; if you
correct the one and leave the other they will
not; do you realize that?

Re: If time goes slower for each twin

<db6e218b-2e13-402b-9690-56dd135b69e6n@googlegroups.com>

  copy mid

https://www.novabbs.com/tech/article-flat.php?id=74941&group=sci.physics.relativity#74941

  copy link   Newsgroups: sci.physics.relativity
X-Received: by 2002:a05:620a:301:: with SMTP id s1mr6672336qkm.771.1639983064772;
Sun, 19 Dec 2021 22:51:04 -0800 (PST)
X-Received: by 2002:ac8:5d8b:: with SMTP id d11mr11152068qtx.434.1639983064661;
Sun, 19 Dec 2021 22:51:04 -0800 (PST)
Path: i2pn2.org!i2pn.org!weretis.net!feeder6.news.weretis.net!news.misty.com!border2.nntp.dca1.giganews.com!nntp.giganews.com!news-out.google.com!nntp.google.com!postnews.google.com!google-groups.googlegroups.com!not-for-mail
Newsgroups: sci.physics.relativity
Date: Sun, 19 Dec 2021 22:51:04 -0800 (PST)
In-Reply-To: <1pkfhui.3xn22u16ilj48N%nospam@de-ster.demon.nl>
Injection-Info: google-groups.googlegroups.com; posting-host=89.206.14.16; posting-account=I3DWzAoAAACOmZUdDcZ-C0PqAZGVsbW0
NNTP-Posting-Host: 89.206.14.16
References: <ade8707c-5779-44d2-9599-c42b4c2cdbf3n@googlegroups.com>
<1pk4fg7.6nqt0j3osf61N%nospam@de-ster.demon.nl> <2088029.irdbgypaU6@PointedEars.de>
<1pk7zpw.1u4709u12ez725N%nospam@de-ster.demon.nl> <jdadncq2e-PC5Cf8nZ2dnUU7_83NnZ2d@giganews.com>
<1pk9vhw.133tjv1sel3x2N%nospam@de-ster.demon.nl> <ec11547f-e187-4a95-96f4-2bb1d4a309b4n@googlegroups.com>
<1pkac7z.1frr3nrtaz7ilN%nospam@de-ster.demon.nl> <nK6dnY3H75YdDib8nZ2dnUU7_83NnZ2d@giganews.com>
<1pkell2.1h3cfauu373o3N%nospam@de-ster.demon.nl> <86f37c21-821d-4d66-9e30-8f487644bb62n@googlegroups.com>
<1pkfhui.3xn22u16ilj48N%nospam@de-ster.demon.nl>
User-Agent: G2/1.0
MIME-Version: 1.0
Message-ID: <db6e218b-2e13-402b-9690-56dd135b69e6n@googlegroups.com>
Subject: Re: If time goes slower for each twin
From: maluwozn...@gmail.com (Maciej Wozniak)
Injection-Date: Mon, 20 Dec 2021 06:51:04 +0000
Content-Type: text/plain; charset="UTF-8"
Content-Transfer-Encoding: quoted-printable
Lines: 45
 by: Maciej Wozniak - Mon, 20 Dec 2021 06:51 UTC

On Sunday, 19 December 2021 at 22:29:03 UTC+1, J. J. Lodder wrote:
> Paparios <mr...@ing.puc.cl> wrote:
>
> > El sábado, 18 de diciembre de 2021 a las 19:32:21 UTC-3, J. J. Lodder:
> > > Tom Roberts <tjrobe...@sbcglobal.net> wrote:
> > >
> > > > On 12/16/21 8:42 AM, J. J. Lodder wrote:
> > > > > Gravitational time dilatation is not a propagation effect.
> > > >
> > > > Hmmmm. See below.
> > > >
> > > > > It is inherent in the gravitational potential being higher or lower.
> > > > > (yes, in Newtonian language)
> > > >
> > > > Hmmmm. See below.
> > > You should stop Hmmm-Hmmming, and think and speak clearly.
> > >
> > > To make sure we talk about the same things I give a simple experiment..
> > > Suppose we have a lab, and several optical clocks.
> > > There is one on the lab bench, and suppose for convenience
> > > that it ticks at 500 THz precisely, so at 5*10^14 Hz.
> > > (with a stability of 10^-18)
> > > Now take a second identical clock,
> > > and place it on a pedestal about one meter higher on the bench.
> > > So with a \Delta\Phi of 10 m^2/s^2. (with c^2 ~= 10^17)
> > >
> >
> > That one meter difference in height, means both clocks are following
> > different paths through spacetime.
> While being both at rest in your room.
> With sufficient purism you can even insist

If you want - you can insist that when a clock
on your shelf desynchronize with the clock
on your desk it means time runs differently
there. And you can present these clocks as
evidence.

> that no two atoms can follow the same path through space-time.
> That is just silly pedantry, without any empirical meaning.

And that's exactly what's what your Shit is, poor
halfbrain.

Re: If time goes slower for each twin

<1pkhc7e.ft6dndkya9ftN%nospam@de-ster.demon.nl>

  copy mid

https://www.novabbs.com/tech/article-flat.php?id=74956&group=sci.physics.relativity#74956

  copy link   Newsgroups: sci.physics.relativity
Path: i2pn2.org!i2pn.org!eternal-september.org!reader02.eternal-september.org!.POSTED!not-for-mail
From: nos...@de-ster.demon.nl (J. J. Lodder)
Newsgroups: sci.physics.relativity
Subject: Re: If time goes slower for each twin
Date: Mon, 20 Dec 2021 11:13:31 +0100
Organization: De Ster
Lines: 180
Message-ID: <1pkhc7e.ft6dndkya9ftN%nospam@de-ster.demon.nl>
References: <ade8707c-5779-44d2-9599-c42b4c2cdbf3n@googlegroups.com> <1pk4fg7.6nqt0j3osf61N%nospam@de-ster.demon.nl> <2088029.irdbgypaU6@PointedEars.de> <1pk7zpw.1u4709u12ez725N%nospam@de-ster.demon.nl> <jdadncq2e-PC5Cf8nZ2dnUU7_83NnZ2d@giganews.com> <1pk9vhw.133tjv1sel3x2N%nospam@de-ster.demon.nl> <ec11547f-e187-4a95-96f4-2bb1d4a309b4n@googlegroups.com> <1pkac7z.1frr3nrtaz7ilN%nospam@de-ster.demon.nl> <nK6dnY3H75YdDib8nZ2dnUU7_83NnZ2d@giganews.com> <1pkell2.1h3cfauu373o3N%nospam@de-ster.demon.nl> <t5udnfZSD4BtUCP8nZ2dnUU7_83NnZ2d@giganews.com> <1pkfhvn.8y3k8armm3o0N%nospam@de-ster.demon.nl> <spo8f4$efk$2@gioia.aioe.org> <1pkgi64.jiwfk5rc4o01N%nospam@de-ster.demon.nl> <spoal7$1dmg$1@gioia.aioe.org>
Reply-To: jjlax32@xs4all.nl (J. J. Lodder)
Injection-Info: reader02.eternal-september.org; posting-host="f60cbf13932189b385757d9f30b3e3d4";
logging-data="15530"; mail-complaints-to="abuse@eternal-september.org"; posting-account="U2FsdGVkX19vdNj9EVfnhTd9gG/Ks8o38gVFqhOzkp8="
User-Agent: MacSOUP/2.8.5 (ea919cf118) (Mac OS 10.10.5)
Cancel-Lock: sha1:QH85LmEiUmtXvT4YOru9I8aZ0ME=
 by: J. J. Lodder - Mon, 20 Dec 2021 10:13 UTC

Python <python@example.invalid> wrote:

> J. J. Lodder wrote:
> > Python <python@example.invalid> wrote:
> >
> >> J. J. Lodder wrote:
> >>> Tom Roberts <tjroberts137@sbcglobal.net> wrote:
> >>>
> >>>> On 12/18/21 4:32 PM, J. J. Lodder wrote:
> >>>>> To make sure we talk about the same things I give a simple experiment.
> >>>>> Suppose we have a lab, and several optical clocks.
> >>>>> There is one on the lab bench, and suppose for convenience
> >>>>> that it ticks at 500 THz precisely, so at 5*10^14 Hz.
> >>>>> (with a stability of 10^-18)
> >>>>> Now take a second identical clock,
> >>>>> and place it on a pedestal about one meter higher on the bench.
> >>>>> So with a \Delta\Phi of 10 m^2/s^2. (with c^2 ~= 10^17)
> >>>>>
> >>>>> By everybody's understanding of general relativity
> >>>>> (and perhaps also by yours)
> >>>>> it will tick at a rate of (1 + 10^-16) THz,
> >>>>> so at 5 x 10^14 + 0.05 Hz.
> >>>>
> >>>> This is wrong. You are presuming your own MISUNDERSTANDING of General
> >>>> Relativity. "Everybody" does not share your misunderstanding, and
> >>>> physicists who understand GR will disagree with your claim. Each clock
> >>>> will tick at 500 THz, because they are identical [#].
> >>>
> >>> So you are in denial of observed reality.
> >>>
> >>>> [#] I am talking about the clocks, AND ONLY THE CLOCKS.
> >>>> So their tick rate is their intrinsic tick rate, which can
> >>>> only be measured in their instantaneously co-movung locally
> >>>> inertial frame (ICLIF). In this case, since the clocks are
> >>>> so very accurate, 1 meter difference in altitude is enough
> >>>> to make each one's ICLIF too small to include the other clock.
> >>>
> >>> The clocks are at rest with respect to each other,
> >>> firmly bolted to the same lab table.
> >>>
> >>> But your whole ICLIF thing is a fundamental error.
> >>> The difference in clock rates is NOT caused by the local gravity.
> >>> It is not a local effect,
> >>> so considering local inertial frames can't deal with it.
> >>> If the geoid changes, so will the difference in clock rates between
> >>> Boulder and Paris change.
> >>>
> >>>>> This difference in frequency is easily demonstrated.
> >>>>> Take light from the upper clock,
> >>>>> and bring it down with mirrors to the lower one,
> >>>>> and let them interfere.
> >>>>> You will see beats with a period of 20 seconds.
> >>>>
> >>>> Sure. But what is the effect of gravity on the light signal, and on the
> >>>> way it is measured? You MUST include that in your analysis.
> >>>
> >>> Of course not. We just see the clocks ticking at different rates.
> >>> Purely empirical, without any need for a theory.
> >>>
> >>>> GR is crystal clear: the beat is caused by the difference in the time
> >>>> component of the metric between the clock and the observing instrument;
> >>>> it appears in the geometrical projection of signal onto instrument. It
> >>>> is NOT due to "identical clocks ticking at different rates" as you think.
> >>>
> >>> That's just your way of denying
> >>> that you see clocks ticking at different rates,
> >>> while standing next to them.
> >>>
> >>>>> Bring light from the lower one up, and idem.
> >>>>> By direct frequency measurement using laser spectroscopy
> >>>>> you can establish that the higher clock is faster,
> >>>>> as predicted by general relativity.
> >>>>
> >>>> GR does not predict the upper clock is "faster", but it does predict the
> >>>> beat, due to the way the interference is observed.
> >>>
> >>> Nice sophistery.
> >>> They differ in rate because they beat, yet they run at the same rate.
> >>> (but only in your mind's eye)
> >>>
> >>>> If you are going to apply "direct frequency measurement" to these
> >>>> clocks, you need to connect each clock to a measuring instrument, and
> >>>> must include the effect of gravitation on the signal between clock and
> >>>> instrument.
> >>>
> >>> You clearly don't know how real life atomic clocks work.
> >>> The counting mechanism is an inherent and integral part
> >>> of the works inside.
> >>>
> >>>> If you do that by connecting each clock to an instrument
> >>>> that is not moved, then the upper clock will be measured to tick faster
> >>>> than the lower clock. If you then analyze the effect of gravity on those
> >>>> signals, and correct the instrument's readings for it, you will find
> >>>> both clocks actually tick at 500 THz [@].
> >>>
> >>> No signals are needed.
> >>> Idealising: both clocks have running digits,
> >>> counting periods of the clock,
> >>> and you can see the higher one getting ahead,
> >>> just by looking at them.
> >>> (from whatever point you want to look)
> >>>
> >>>> [@] Again I remind you that this English phrase refers
> >>>> just to the clocks, so it refers to their intrinsic
> >>>> tick rate, measured in their ICLIF.
> >>>
> >>> Your linguistic pedantry is wrong too, see elsewhere.
> >>>
> >>>>> In principle (and nowadays almost in practice)
> >>>>> you can add a counter to each clock
> >>>>> that counts the individual wave crests directly.
> >>>>> (like on a Cesium clock)
> >>>>> Again, by just reading the digits you will see
> >>>>> that the higher clock runs faster,
> >>>>> without the need for making any connection between the clocks.
> >>>>
> >>>> For a given interval in the lab, you will see the upper clock
> >>>> accumulates more ticks than the lower clock. GR models this as due to
> >>>> their different paths through spacetime, not due to any difference in
> >>>> tick rates. This is DIFFERENT from the experiment discussed above (which
> >>>> involves tick rates).
> >>>>
> >>>> What you see with your eyes here is MIXING the ICLIFs of the two clocks,
> >>>> and you have ignored that, making your conclusion worthless.
> >>>
> >>> That is just your unnecessary, and incorrect overhead.
> >>>
> >>>>> Now every 'ordinary' person, and almost? all physicists,
> >>>>> will say that what we see here are two clocks running at slightly
> >>>>> different rates. (as predicted by general relativity)
> >>>>
> >>>> Your claim is WRONG. Physicists who understand GR will disagree with you
> >>>> and agree with what I said above. For the simple reason that I am
> >>>> describing how GR models this, and you are NOT.
> >>>
> >>> People who really do this kind of thing strongly disagree with you.
> >>>
> >>>>> Only you insist that the faster clock is not 'really' running faster.
> >>>>
> >>>> This is not "only me", it is how GR models these two different
> >>>>situations.
> >>>>
> >>>> GR is more complicated than you think, and much of what you think about
> >>>> it is WRONG. Your problem, not mine.
> >>>
> >>> OK, so you have misunderstood what general relativity
> >>> has to say about it, and you remain in denial of observed reality.
> >>>
> >>> No more to be said, I guess,
> >>>
> >>> Jan
> >>
> >>
> >> You should consider thinking about this, Jan. Tom is right.
> >
> > Nope. Clocks at different altitudes do tick at different rates,
> > and anyone who has the right tools can measure that.
> > You might want to search on 'relativistic geodesy',
> > or 'chronometric levelling' for some of the facts.
> >
> > We will soon be able to map out the geoid
> > to a new level of accuracy,
> > by measuring differences in clock rates,
> > all over the planet,
> >
> > Jan
>
>
> Jan, think about the meaning of words.


Click here to read the complete article
Re: If time goes slower for each twin

<1pkhc9l.1f5je0511gjz9tN%nospam@de-ster.demon.nl>

  copy mid

https://www.novabbs.com/tech/article-flat.php?id=74957&group=sci.physics.relativity#74957

  copy link   Newsgroups: sci.physics.relativity
Path: i2pn2.org!i2pn.org!eternal-september.org!reader02.eternal-september.org!.POSTED!not-for-mail
From: nos...@de-ster.demon.nl (J. J. Lodder)
Newsgroups: sci.physics.relativity
Subject: Re: If time goes slower for each twin
Date: Mon, 20 Dec 2021 11:13:32 +0100
Organization: De Ster
Lines: 69
Message-ID: <1pkhc9l.1f5je0511gjz9tN%nospam@de-ster.demon.nl>
References: <ade8707c-5779-44d2-9599-c42b4c2cdbf3n@googlegroups.com> <1pk4fg7.6nqt0j3osf61N%nospam@de-ster.demon.nl> <2088029.irdbgypaU6@PointedEars.de> <1pk7zpw.1u4709u12ez725N%nospam@de-ster.demon.nl> <jdadncq2e-PC5Cf8nZ2dnUU7_83NnZ2d@giganews.com> <1pk9vhw.133tjv1sel3x2N%nospam@de-ster.demon.nl> <ec11547f-e187-4a95-96f4-2bb1d4a309b4n@googlegroups.com> <1pkac7z.1frr3nrtaz7ilN%nospam@de-ster.demon.nl> <nK6dnY3H75YdDib8nZ2dnUU7_83NnZ2d@giganews.com> <1pkell2.1h3cfauu373o3N%nospam@de-ster.demon.nl> <t5udnfZSD4BtUCP8nZ2dnUU7_83NnZ2d@giganews.com> <1pkfhvn.8y3k8armm3o0N%nospam@de-ster.demon.nl> <36e4e94f-48e6-43a0-baae-e8f3e3865478n@googlegroups.com>
Reply-To: jjlax32@xs4all.nl (J. J. Lodder)
Injection-Info: reader02.eternal-september.org; posting-host="f60cbf13932189b385757d9f30b3e3d4";
logging-data="15530"; mail-complaints-to="abuse@eternal-september.org"; posting-account="U2FsdGVkX190+ysL82rXwPzu5g6MyFdMN2ijwQDKQsg="
User-Agent: MacSOUP/2.8.5 (ea919cf118) (Mac OS 10.10.5)
Cancel-Lock: sha1:H1ONdN1WyqV8rpWIwX2zgHNhlEU=
 by: J. J. Lodder - Mon, 20 Dec 2021 10:13 UTC

RichD <r_delaney2001@yahoo.com> wrote:

> On December 19, J. J. Lodder wrote:
> >>> Suppose we have a lab, and several optical clocks.
> >>> There is one on the lab bench, and suppose
> >>> that it ticks at 500 THz precisely, so at 5*10^14 Hz
> >>> Now take a second identical clock,
> >>> and place it on a pedestal about one meter higher on the bench.
> >>> So with a \Delta\Phi of 10 m^2/s^2
> >>> By everybody's understanding of general relativity
> >>> it will tick at a rate of (1 + 10^-16) THz,
> >>> so at 5 x 10^14 + 0.05 Hz.
> >
> >> [#] I am talking about the clocks, AND ONLY THE CLOCKS.
> >> So their tick rate is their intrinsic tick rate, which can
> >> only be measured in their instantaneously co-movung locally
> >> inertial frame (ICLIF). In this case, since the clocks are
> >> so very accurate, 1 meter difference in altitude is enough
> >> to make each one's ICLIF too small to include the other clock.
> >
> > The clocks are at rest with respect to each other,
> > firmly bolted to the same lab table.
> > But your whole ICLIF thing is a fundamental error.
> > The difference in clock rates is NOT caused by the local gravity.
>
> Now let me get this straight:
> One clock is at a higher altitude, by one meter.
> The higher altitude unit ticks faster, as observed in the lab.
> General relativity predicts this, by "everybody's understanding".
> But it's NOT caused by a difference in local gravity.

Correct. It is -not- a local effect.
It depends on the difference in Newtonian potential
between the high and the low clock,
so NOT on the local value of (small) g. [1]

So it depends on the whole mass distribution of the Earth.
That is precisely why measuring it precisely will be very useful,
in the near future.

> "You humans are quite illogical."
> - Mr. Spock

He must have been reading spr(elativity) instead of spr(esearch)
He can't be blamed for it, I guess, since esearch is almost dead.
Oh well, he could read back issues.

> > OK, so you have misunderstood what general relativity
> > has to say about it, and you remain in denial of observed reality.
> > No more to be said, I guess,
>
> I say it's leprechauns - if it isn't due to gravity, leprechauns are
> the only thing left -

But it -is- due to gravity,
but not in the way some people misunderstand,

Jan

[1] The same is true for Pound-Rebka,
about which the same misunderstandings exist.
What they really measure is \Delta\Phi too.

The formula for the prediction for Pound-Rebka
is usually given in terms of g \Delta h.
One should realise that g = d\Phi/dh,
so g \Delta h is just \Delta\Phi.

Re: If time goes slower for each twin

<30a3f9ce-df95-4cec-9ae4-d08e6e9f2d30n@googlegroups.com>

  copy mid

https://www.novabbs.com/tech/article-flat.php?id=74963&group=sci.physics.relativity#74963

  copy link   Newsgroups: sci.physics.relativity
X-Received: by 2002:ad4:58cf:: with SMTP id dh15mr12129925qvb.125.1639995748893;
Mon, 20 Dec 2021 02:22:28 -0800 (PST)
X-Received: by 2002:ac8:5d8b:: with SMTP id d11mr11551277qtx.434.1639995748759;
Mon, 20 Dec 2021 02:22:28 -0800 (PST)
Path: i2pn2.org!i2pn.org!weretis.net!feeder6.news.weretis.net!news.misty.com!border2.nntp.dca1.giganews.com!border1.nntp.dca1.giganews.com!nntp.giganews.com!news-out.google.com!nntp.google.com!postnews.google.com!google-groups.googlegroups.com!not-for-mail
Newsgroups: sci.physics.relativity
Date: Mon, 20 Dec 2021 02:22:28 -0800 (PST)
In-Reply-To: <1pkhc7e.ft6dndkya9ftN%nospam@de-ster.demon.nl>
Injection-Info: google-groups.googlegroups.com; posting-host=89.206.14.16; posting-account=I3DWzAoAAACOmZUdDcZ-C0PqAZGVsbW0
NNTP-Posting-Host: 89.206.14.16
References: <ade8707c-5779-44d2-9599-c42b4c2cdbf3n@googlegroups.com>
<1pk4fg7.6nqt0j3osf61N%nospam@de-ster.demon.nl> <2088029.irdbgypaU6@PointedEars.de>
<1pk7zpw.1u4709u12ez725N%nospam@de-ster.demon.nl> <jdadncq2e-PC5Cf8nZ2dnUU7_83NnZ2d@giganews.com>
<1pk9vhw.133tjv1sel3x2N%nospam@de-ster.demon.nl> <ec11547f-e187-4a95-96f4-2bb1d4a309b4n@googlegroups.com>
<1pkac7z.1frr3nrtaz7ilN%nospam@de-ster.demon.nl> <nK6dnY3H75YdDib8nZ2dnUU7_83NnZ2d@giganews.com>
<1pkell2.1h3cfauu373o3N%nospam@de-ster.demon.nl> <t5udnfZSD4BtUCP8nZ2dnUU7_83NnZ2d@giganews.com>
<1pkfhvn.8y3k8armm3o0N%nospam@de-ster.demon.nl> <spo8f4$efk$2@gioia.aioe.org>
<1pkgi64.jiwfk5rc4o01N%nospam@de-ster.demon.nl> <spoal7$1dmg$1@gioia.aioe.org>
<1pkhc7e.ft6dndkya9ftN%nospam@de-ster.demon.nl>
User-Agent: G2/1.0
MIME-Version: 1.0
Message-ID: <30a3f9ce-df95-4cec-9ae4-d08e6e9f2d30n@googlegroups.com>
Subject: Re: If time goes slower for each twin
From: maluwozn...@gmail.com (Maciej Wozniak)
Injection-Date: Mon, 20 Dec 2021 10:22:28 +0000
Content-Type: text/plain; charset="UTF-8"
Lines: 178
 by: Maciej Wozniak - Mon, 20 Dec 2021 10:22 UTC

On Monday, 20 December 2021 at 11:13:35 UTC+1, J. J. Lodder wrote:
> Python <pyt...@example.invalid> wrote:
>
> > J. J. Lodder wrote:
> > > Python <pyt...@example.invalid> wrote:
> > >
> > >> J. J. Lodder wrote:
> > >>> Tom Roberts <tjrobe...@sbcglobal.net> wrote:
> > >>>
> > >>>> On 12/18/21 4:32 PM, J. J. Lodder wrote:
> > >>>>> To make sure we talk about the same things I give a simple experiment.
> > >>>>> Suppose we have a lab, and several optical clocks.
> > >>>>> There is one on the lab bench, and suppose for convenience
> > >>>>> that it ticks at 500 THz precisely, so at 5*10^14 Hz.
> > >>>>> (with a stability of 10^-18)
> > >>>>> Now take a second identical clock,
> > >>>>> and place it on a pedestal about one meter higher on the bench.
> > >>>>> So with a \Delta\Phi of 10 m^2/s^2. (with c^2 ~= 10^17)
> > >>>>>
> > >>>>> By everybody's understanding of general relativity
> > >>>>> (and perhaps also by yours)
> > >>>>> it will tick at a rate of (1 + 10^-16) THz,
> > >>>>> so at 5 x 10^14 + 0.05 Hz.
> > >>>>
> > >>>> This is wrong. You are presuming your own MISUNDERSTANDING of General
> > >>>> Relativity. "Everybody" does not share your misunderstanding, and
> > >>>> physicists who understand GR will disagree with your claim. Each clock
> > >>>> will tick at 500 THz, because they are identical [#].
> > >>>
> > >>> So you are in denial of observed reality.
> > >>>
> > >>>> [#] I am talking about the clocks, AND ONLY THE CLOCKS.
> > >>>> So their tick rate is their intrinsic tick rate, which can
> > >>>> only be measured in their instantaneously co-movung locally
> > >>>> inertial frame (ICLIF). In this case, since the clocks are
> > >>>> so very accurate, 1 meter difference in altitude is enough
> > >>>> to make each one's ICLIF too small to include the other clock.
> > >>>
> > >>> The clocks are at rest with respect to each other,
> > >>> firmly bolted to the same lab table.
> > >>>
> > >>> But your whole ICLIF thing is a fundamental error.
> > >>> The difference in clock rates is NOT caused by the local gravity.
> > >>> It is not a local effect,
> > >>> so considering local inertial frames can't deal with it.
> > >>> If the geoid changes, so will the difference in clock rates between
> > >>> Boulder and Paris change.
> > >>>
> > >>>>> This difference in frequency is easily demonstrated.
> > >>>>> Take light from the upper clock,
> > >>>>> and bring it down with mirrors to the lower one,
> > >>>>> and let them interfere.
> > >>>>> You will see beats with a period of 20 seconds.
> > >>>>
> > >>>> Sure. But what is the effect of gravity on the light signal, and on the
> > >>>> way it is measured? You MUST include that in your analysis.
> > >>>
> > >>> Of course not. We just see the clocks ticking at different rates.
> > >>> Purely empirical, without any need for a theory.
> > >>>
> > >>>> GR is crystal clear: the beat is caused by the difference in the time
> > >>>> component of the metric between the clock and the observing instrument;
> > >>>> it appears in the geometrical projection of signal onto instrument. It
> > >>>> is NOT due to "identical clocks ticking at different rates" as you think.
> > >>>
> > >>> That's just your way of denying
> > >>> that you see clocks ticking at different rates,
> > >>> while standing next to them.
> > >>>
> > >>>>> Bring light from the lower one up, and idem.
> > >>>>> By direct frequency measurement using laser spectroscopy
> > >>>>> you can establish that the higher clock is faster,
> > >>>>> as predicted by general relativity.
> > >>>>
> > >>>> GR does not predict the upper clock is "faster", but it does predict the
> > >>>> beat, due to the way the interference is observed.
> > >>>
> > >>> Nice sophistery.
> > >>> They differ in rate because they beat, yet they run at the same rate.
> > >>> (but only in your mind's eye)
> > >>>
> > >>>> If you are going to apply "direct frequency measurement" to these
> > >>>> clocks, you need to connect each clock to a measuring instrument, and
> > >>>> must include the effect of gravitation on the signal between clock and
> > >>>> instrument.
> > >>>
> > >>> You clearly don't know how real life atomic clocks work.
> > >>> The counting mechanism is an inherent and integral part
> > >>> of the works inside.
> > >>>
> > >>>> If you do that by connecting each clock to an instrument
> > >>>> that is not moved, then the upper clock will be measured to tick faster
> > >>>> than the lower clock. If you then analyze the effect of gravity on those
> > >>>> signals, and correct the instrument's readings for it, you will find
> > >>>> both clocks actually tick at 500 THz [@].
> > >>>
> > >>> No signals are needed.
> > >>> Idealising: both clocks have running digits,
> > >>> counting periods of the clock,
> > >>> and you can see the higher one getting ahead,
> > >>> just by looking at them.
> > >>> (from whatever point you want to look)
> > >>>
> > >>>> [@] Again I remind you that this English phrase refers
> > >>>> just to the clocks, so it refers to their intrinsic
> > >>>> tick rate, measured in their ICLIF.
> > >>>
> > >>> Your linguistic pedantry is wrong too, see elsewhere.
> > >>>
> > >>>>> In principle (and nowadays almost in practice)
> > >>>>> you can add a counter to each clock
> > >>>>> that counts the individual wave crests directly.
> > >>>>> (like on a Cesium clock)
> > >>>>> Again, by just reading the digits you will see
> > >>>>> that the higher clock runs faster,
> > >>>>> without the need for making any connection between the clocks.
> > >>>>
> > >>>> For a given interval in the lab, you will see the upper clock
> > >>>> accumulates more ticks than the lower clock. GR models this as due to
> > >>>> their different paths through spacetime, not due to any difference in
> > >>>> tick rates. This is DIFFERENT from the experiment discussed above (which
> > >>>> involves tick rates).
> > >>>>
> > >>>> What you see with your eyes here is MIXING the ICLIFs of the two clocks,
> > >>>> and you have ignored that, making your conclusion worthless.
> > >>>
> > >>> That is just your unnecessary, and incorrect overhead.
> > >>>
> > >>>>> Now every 'ordinary' person, and almost? all physicists,
> > >>>>> will say that what we see here are two clocks running at slightly
> > >>>>> different rates. (as predicted by general relativity)
> > >>>>
> > >>>> Your claim is WRONG. Physicists who understand GR will disagree with you
> > >>>> and agree with what I said above. For the simple reason that I am
> > >>>> describing how GR models this, and you are NOT.
> > >>>
> > >>> People who really do this kind of thing strongly disagree with you.
> > >>>
> > >>>>> Only you insist that the faster clock is not 'really' running faster.
> > >>>>
> > >>>> This is not "only me", it is how GR models these two different
> > >>>>situations.
> > >>>>
> > >>>> GR is more complicated than you think, and much of what you think about
> > >>>> it is WRONG. Your problem, not mine.
> > >>>
> > >>> OK, so you have misunderstood what general relativity
> > >>> has to say about it, and you remain in denial of observed reality.
> > >>>
> > >>> No more to be said, I guess,
> > >>>
> > >>> Jan
> > >>
> > >>
> > >> You should consider thinking about this, Jan. Tom is right.
> > >
> > > Nope. Clocks at different altitudes do tick at different rates,
> > > and anyone who has the right tools can measure that.
> > > You might want to search on 'relativistic geodesy',
> > > or 'chronometric levelling' for some of the facts.
> > >
> > > We will soon be able to map out the geoid
> > > to a new level of accuracy,
> > > by measuring differences in clock rates,
> > > all over the planet,
> > >
> > > Jan
> >
> >
> > Jan, think about the meaning of words.
> Physics is about all that, and only about all that,
> which does not depend on the words we use to describe it.


Click here to read the complete article
Re: If time goes slower for each twin

<68e8acd3-4e29-46a5-bdd6-75dc5f060cb8n@googlegroups.com>

  copy mid

https://www.novabbs.com/tech/article-flat.php?id=74971&group=sci.physics.relativity#74971

  copy link   Newsgroups: sci.physics.relativity
X-Received: by 2002:a05:622a:4ce:: with SMTP id q14mr12550129qtx.627.1640009743545;
Mon, 20 Dec 2021 06:15:43 -0800 (PST)
X-Received: by 2002:a05:6214:411a:: with SMTP id kc26mr12686847qvb.113.1640009743302;
Mon, 20 Dec 2021 06:15:43 -0800 (PST)
Path: i2pn2.org!i2pn.org!weretis.net!feeder6.news.weretis.net!news.misty.com!border2.nntp.dca1.giganews.com!nntp.giganews.com!news-out.google.com!nntp.google.com!postnews.google.com!google-groups.googlegroups.com!not-for-mail
Newsgroups: sci.physics.relativity
Date: Mon, 20 Dec 2021 06:15:43 -0800 (PST)
In-Reply-To: <1pkfhui.3xn22u16ilj48N%nospam@de-ster.demon.nl>
Injection-Info: google-groups.googlegroups.com; posting-host=2800:150:125:359e:5dbc:bb34:dd43:4432;
posting-account=KA67VQoAAAABNtRUVf2Wh-jHtkEfmXxT
NNTP-Posting-Host: 2800:150:125:359e:5dbc:bb34:dd43:4432
References: <ade8707c-5779-44d2-9599-c42b4c2cdbf3n@googlegroups.com>
<1pk4fg7.6nqt0j3osf61N%nospam@de-ster.demon.nl> <2088029.irdbgypaU6@PointedEars.de>
<1pk7zpw.1u4709u12ez725N%nospam@de-ster.demon.nl> <jdadncq2e-PC5Cf8nZ2dnUU7_83NnZ2d@giganews.com>
<1pk9vhw.133tjv1sel3x2N%nospam@de-ster.demon.nl> <ec11547f-e187-4a95-96f4-2bb1d4a309b4n@googlegroups.com>
<1pkac7z.1frr3nrtaz7ilN%nospam@de-ster.demon.nl> <nK6dnY3H75YdDib8nZ2dnUU7_83NnZ2d@giganews.com>
<1pkell2.1h3cfauu373o3N%nospam@de-ster.demon.nl> <86f37c21-821d-4d66-9e30-8f487644bb62n@googlegroups.com>
<1pkfhui.3xn22u16ilj48N%nospam@de-ster.demon.nl>
User-Agent: G2/1.0
MIME-Version: 1.0
Message-ID: <68e8acd3-4e29-46a5-bdd6-75dc5f060cb8n@googlegroups.com>
Subject: Re: If time goes slower for each twin
From: mri...@ing.puc.cl (Paparios)
Injection-Date: Mon, 20 Dec 2021 14:15:43 +0000
Content-Type: text/plain; charset="UTF-8"
Content-Transfer-Encoding: quoted-printable
Lines: 76
 by: Paparios - Mon, 20 Dec 2021 14:15 UTC

El domingo, 19 de diciembre de 2021 a las 18:29:03 UTC-3, J. J. Lodder escribió:
> Paparios <mr...@ing.puc.cl> wrote:
>
> > El sábado, 18 de diciembre de 2021 a las 19:32:21 UTC-3, J. J. Lodder:
> > > Tom Roberts <tjrobe...@sbcglobal.net> wrote:
> > >
> > > > On 12/16/21 8:42 AM, J. J. Lodder wrote:
> > > > > Gravitational time dilatation is not a propagation effect.
> > > >
> > > > Hmmmm. See below.
> > > >
> > > > > It is inherent in the gravitational potential being higher or lower.
> > > > > (yes, in Newtonian language)
> > > >
> > > > Hmmmm. See below.
> > > You should stop Hmmm-Hmmming, and think and speak clearly.
> > >
> > > To make sure we talk about the same things I give a simple experiment..
> > > Suppose we have a lab, and several optical clocks.
> > > There is one on the lab bench, and suppose for convenience
> > > that it ticks at 500 THz precisely, so at 5*10^14 Hz.
> > > (with a stability of 10^-18)
> > > Now take a second identical clock,
> > > and place it on a pedestal about one meter higher on the bench.
> > > So with a \Delta\Phi of 10 m^2/s^2. (with c^2 ~= 10^17)
> > >
> >
> > That one meter difference in height, means both clocks are following
> > different paths through spacetime.
> While being both at rest in your room.
> With sufficient purism you can even insist
> that no two atoms can follow the same path through space-time.
> That is just silly pedantry, without any empirical meaning.
> > > By everybody's understanding of general relativity
> > > (and perhaps also by yours)
> > > it will tick at a rate of (1 + 10^-16) THz,
> > > so at 5 x 10^14 + 0.05 Hz.
> > > This difference in frequency is easily demonstrated.
> > > Take light from the upper clock,
> > > and bring it down with mirrors to the lower one,
> > > and let them interfere.
> > > You will see beats with a period of 20 seconds.
> > > Bring light from the lower one up, and idem.
> > > By direct frequency measurement using laser spectroscopy
> > > you can establish that the higher clock is faster,
> > > as predicted by general relativity.
> >
> > Sure, but in that case you are sending signals from the higher clock to
> > the lower clock and then making the comparison there. In order to measure
> > the time interval between them, the receiving clock must project that
> > displacement 4-vector onto itself; that is a dot product involving the
> > metric at the receiving clock's location.
> No need to, but you snipped that.
> In principle each clock can just count its ticks,
> and you can see the counters diverge.
>
> Jan

Actually, that is a much more complex task. One of the best tests on gravitational time dilation was performed using optical lattice clocks in Tokio. The experiment is decribed in the paper "Takamoto et al. (2020) Test of general relativity by a pair of transportable optical lattice clocks. Nat Photonics. doi: 10.1038/s41566-020-0619-8". You can read the paper at the following location: https://www.nature.com/articles/s41566-020-0619-8.pdf

They need to check for around 10 centimeters height variations of the Tokio Tower (452 meters) due to temperature over the 6 months test period. Obviously, they are measuring the elapsed time of each clock and comparing their time difference to verify equation 1 in the paper.

Re: If time goes slower for each twin

<31f83c90-e125-49a6-93ad-09bf2334f132n@googlegroups.com>

  copy mid

https://www.novabbs.com/tech/article-flat.php?id=74973&group=sci.physics.relativity#74973

  copy link   Newsgroups: sci.physics.relativity
X-Received: by 2002:a05:622a:1654:: with SMTP id y20mr2539695qtj.374.1640015297457;
Mon, 20 Dec 2021 07:48:17 -0800 (PST)
X-Received: by 2002:a05:620a:1929:: with SMTP id bj41mr4939792qkb.34.1640015297333;
Mon, 20 Dec 2021 07:48:17 -0800 (PST)
Path: i2pn2.org!i2pn.org!weretis.net!feeder6.news.weretis.net!news.misty.com!border2.nntp.dca1.giganews.com!nntp.giganews.com!news-out.google.com!nntp.google.com!postnews.google.com!google-groups.googlegroups.com!not-for-mail
Newsgroups: sci.physics.relativity
Date: Mon, 20 Dec 2021 07:48:17 -0800 (PST)
In-Reply-To: <68e8acd3-4e29-46a5-bdd6-75dc5f060cb8n@googlegroups.com>
Injection-Info: google-groups.googlegroups.com; posting-host=83.8.71.212; posting-account=I3DWzAoAAACOmZUdDcZ-C0PqAZGVsbW0
NNTP-Posting-Host: 83.8.71.212
References: <ade8707c-5779-44d2-9599-c42b4c2cdbf3n@googlegroups.com>
<1pk4fg7.6nqt0j3osf61N%nospam@de-ster.demon.nl> <2088029.irdbgypaU6@PointedEars.de>
<1pk7zpw.1u4709u12ez725N%nospam@de-ster.demon.nl> <jdadncq2e-PC5Cf8nZ2dnUU7_83NnZ2d@giganews.com>
<1pk9vhw.133tjv1sel3x2N%nospam@de-ster.demon.nl> <ec11547f-e187-4a95-96f4-2bb1d4a309b4n@googlegroups.com>
<1pkac7z.1frr3nrtaz7ilN%nospam@de-ster.demon.nl> <nK6dnY3H75YdDib8nZ2dnUU7_83NnZ2d@giganews.com>
<1pkell2.1h3cfauu373o3N%nospam@de-ster.demon.nl> <86f37c21-821d-4d66-9e30-8f487644bb62n@googlegroups.com>
<1pkfhui.3xn22u16ilj48N%nospam@de-ster.demon.nl> <68e8acd3-4e29-46a5-bdd6-75dc5f060cb8n@googlegroups.com>
User-Agent: G2/1.0
MIME-Version: 1.0
Message-ID: <31f83c90-e125-49a6-93ad-09bf2334f132n@googlegroups.com>
Subject: Re: If time goes slower for each twin
From: maluwozn...@gmail.com (Maciej Wozniak)
Injection-Date: Mon, 20 Dec 2021 15:48:17 +0000
Content-Type: text/plain; charset="UTF-8"
Content-Transfer-Encoding: quoted-printable
Lines: 73
 by: Maciej Wozniak - Mon, 20 Dec 2021 15:48 UTC

On Monday, 20 December 2021 at 15:15:44 UTC+1, Paparios wrote:
> El domingo, 19 de diciembre de 2021 a las 18:29:03 UTC-3, J. J. Lodder escribió:
> > Paparios <mr...@ing.puc.cl> wrote:
> >
> > > El sábado, 18 de diciembre de 2021 a las 19:32:21 UTC-3, J. J. Lodder:
> > > > Tom Roberts <tjrobe...@sbcglobal.net> wrote:
> > > >
> > > > > On 12/16/21 8:42 AM, J. J. Lodder wrote:
> > > > > > Gravitational time dilatation is not a propagation effect.
> > > > >
> > > > > Hmmmm. See below.
> > > > >
> > > > > > It is inherent in the gravitational potential being higher or lower.
> > > > > > (yes, in Newtonian language)
> > > > >
> > > > > Hmmmm. See below.
> > > > You should stop Hmmm-Hmmming, and think and speak clearly.
> > > >
> > > > To make sure we talk about the same things I give a simple experiment.
> > > > Suppose we have a lab, and several optical clocks.
> > > > There is one on the lab bench, and suppose for convenience
> > > > that it ticks at 500 THz precisely, so at 5*10^14 Hz.
> > > > (with a stability of 10^-18)
> > > > Now take a second identical clock,
> > > > and place it on a pedestal about one meter higher on the bench.
> > > > So with a \Delta\Phi of 10 m^2/s^2. (with c^2 ~= 10^17)
> > > >
> > >
> > > That one meter difference in height, means both clocks are following
> > > different paths through spacetime.
> > While being both at rest in your room.
> > With sufficient purism you can even insist
> > that no two atoms can follow the same path through space-time.
> > That is just silly pedantry, without any empirical meaning.
> > > > By everybody's understanding of general relativity
> > > > (and perhaps also by yours)
> > > > it will tick at a rate of (1 + 10^-16) THz,
> > > > so at 5 x 10^14 + 0.05 Hz.
> > > > This difference in frequency is easily demonstrated.
> > > > Take light from the upper clock,
> > > > and bring it down with mirrors to the lower one,
> > > > and let them interfere.
> > > > You will see beats with a period of 20 seconds.
> > > > Bring light from the lower one up, and idem.
> > > > By direct frequency measurement using laser spectroscopy
> > > > you can establish that the higher clock is faster,
> > > > as predicted by general relativity.
> > >
> > > Sure, but in that case you are sending signals from the higher clock to
> > > the lower clock and then making the comparison there. In order to measure
> > > the time interval between them, the receiving clock must project that
> > > displacement 4-vector onto itself; that is a dot product involving the
> > > metric at the receiving clock's location.
> > No need to, but you snipped that.
> > In principle each clock can just count its ticks,
> > and you can see the counters diverge.
> >
> > Jan
> Actually, that is a much more complex task. One of the best tests on gravitational time dilation was performed using optical lattice clocks in Tokio..

In the meantime in the real world, however, forbidden
by your moronic religion TAI keep measuring t'=t, just
like all serious clocks always did.

Re: If time goes slower for each twin

<spqccv$egs$3@gioia.aioe.org>

  copy mid

https://www.novabbs.com/tech/article-flat.php?id=74979&group=sci.physics.relativity#74979

  copy link   Newsgroups: sci.physics.relativity
Path: i2pn2.org!i2pn.org!aioe.org!Lenw9N2TgqlbGNOh+3DBoA.user.46.165.242.75.POSTED!not-for-mail
From: ert...@vbmv.ty (Wade Evers)
Newsgroups: sci.physics.relativity
Subject: Re: If time goes slower for each twin
Date: Mon, 20 Dec 2021 16:52:48 -0000 (UTC)
Organization: Aioe.org NNTP Server
Message-ID: <spqccv$egs$3@gioia.aioe.org>
References: <ade8707c-5779-44d2-9599-c42b4c2cdbf3n@googlegroups.com>
<1pk4fg7.6nqt0j3osf61N%nospam@de-ster.demon.nl>
<2088029.irdbgypaU6@PointedEars.de>
<1pk7zpw.1u4709u12ez725N%nospam@de-ster.demon.nl>
<jdadncq2e-PC5Cf8nZ2dnUU7_83NnZ2d@giganews.com>
<1pk9vhw.133tjv1sel3x2N%nospam@de-ster.demon.nl>
<ec11547f-e187-4a95-96f4-2bb1d4a309b4n@googlegroups.com>
<1pkac7z.1frr3nrtaz7ilN%nospam@de-ster.demon.nl>
<nK6dnY3H75YdDib8nZ2dnUU7_83NnZ2d@giganews.com>
<1pkell2.1h3cfauu373o3N%nospam@de-ster.demon.nl>
<t5udnfZSD4BtUCP8nZ2dnUU7_83NnZ2d@giganews.com>
<1pkfhvn.8y3k8armm3o0N%nospam@de-ster.demon.nl>
<spo8f4$efk$2@gioia.aioe.org>
<1pkgi64.jiwfk5rc4o01N%nospam@de-ster.demon.nl>
<spoal7$1dmg$1@gioia.aioe.org>
<1pkhc7e.ft6dndkya9ftN%nospam@de-ster.demon.nl>
<30a3f9ce-df95-4cec-9ae4-d08e6e9f2d30n@googlegroups.com>
Mime-Version: 1.0
Content-Type: text/plain; charset=UTF-8
Content-Transfer-Encoding: 8bit
Injection-Info: gioia.aioe.org; logging-data="14876"; posting-host="Lenw9N2TgqlbGNOh+3DBoA.user.gioia.aioe.org"; mail-complaints-to="abuse@aioe.org";
User-Agent: Mozilla/5.0 (Windows NT 10.0; Win64; x64; rv:78.0) Gecko/20100101
Thunderbird/78.7.1
X-Notice: Filtered by postfilter v. 0.9.2
 by: Wade Evers - Mon, 20 Dec 2021 16:52 UTC

Maciej Wozniak wrote:

> On Monday, 20 December 2021 at 11:13:35 UTC+1, J. J. Lodder wrote:
>> Python <pyt...@example.invalid> wrote:
>> > Jan, think about the meaning of words.
>> Physics is about all that, and only about all that, which does not
>> depend on the words we use to describe it.
>
> If it really was, it would be about nothing, poor fanatic idiot.

please don't talk with them. They are unemployable imbeciles. To you it's
obvious, but they are pretending something else.

Re: If time goes slower for each twin

<tei1sg5t1ffmbimprks38hm016q3ap7sqj@4ax.com>

  copy mid

https://www.novabbs.com/tech/article-flat.php?id=74992&group=sci.physics.relativity#74992

  copy link   Newsgroups: sci.physics.relativity
Path: i2pn2.org!i2pn.org!weretis.net!feeder6.news.weretis.net!news.misty.com!border2.nntp.dca1.giganews.com!nntp.giganews.com!buffer2.nntp.dca1.giganews.com!buffer1.nntp.dca1.giganews.com!nntp.earthlink.com!news.earthlink.com.POSTED!not-for-mail
NNTP-Posting-Date: Mon, 20 Dec 2021 12:46:46 -0600
From: starma...@ix.netcom.com (The Starmaker)
Newsgroups: sci.physics.relativity
Subject: Re: If time goes slower for each twin
Date: Mon, 20 Dec 2021 10:46:47 -0800
Message-ID: <tei1sg5t1ffmbimprks38hm016q3ap7sqj@4ax.com>
References: <ade8707c-5779-44d2-9599-c42b4c2cdbf3n@googlegroups.com>
X-Newsreader: Forte Agent 5.00/32.1171
MIME-Version: 1.0
Content-Type: text/plain; charset=us-ascii
Content-Transfer-Encoding: 7bit
X-Antivirus: Avast (VPS 211220-0, 12/19/2021), Outbound message
X-Antivirus-Status: Clean
Lines: 91
X-Usenet-Provider: http://www.giganews.com
NNTP-Posting-Host: 108.219.229.47
X-Trace: sv3-b9kNF31+tiHYHRFT+ou7sm8H28+ORh0A9eUScJmoiSWp0X9TekvfRsYlatwAIXidMzr7bRufTeGIVpt!q1mlY9X8+Y+Vct2DqbwWAPQXoC2eI6RGQX1GQL6L3YJuU8LhTMJpxmqEB1ks4PSIpDRyvQtacxX3!k1L0VTzb7GV81MbqUIccB62jAg3lnBY=
X-Abuse-and-DMCA-Info: Please be sure to forward a copy of ALL headers
X-Abuse-and-DMCA-Info: Otherwise we will be unable to process your complaint properly
X-Postfilter: 1.3.40
X-Original-Bytes: 2743
 by: The Starmaker - Mon, 20 Dec 2021 18:46 UTC

On Sat, 11 Dec 2021 12:49:41 -0800 (PST), "mitchr...@gmail.com"
<mitchrae3323@gmail.com> wrote:

>would they not have equal clocks
>by relative separation?

Yous people don't seem to have any understanding of what Time is...

Time and Space existed 'Before' the universe was created.

It's in the 'Code'. you just need to decipher it correctly.

The code is...'In the beginning God created the heavens and the
earth.'

"'In the beginning..." means,

Beginning:
"the point in time or space at which something starts."

The "beginning" is a point in time, not the beginning of time.

It is at which something starts, like for example...a universe.

Before that point in time you have Time.

Pick a point ' in' Time and that is when you can make 'something' to
start..like a universe.

Or pick an earlier point in time.

So, that point in Time would be like...14 billion years ago.

Now, don't tell me yous don't understand what "the point in time"
means.

Just take your little finger and point to where you want to start
something.

That's the beginning.

"the point in time or space at which something starts."

It is simply the first frame of the movie...

16mm

the part that gets exposed

Have you even seen an unexposed 16mm film strip?

comes in a container

nothing on it....

lights!

action!

The Starmaker

--
The Starmaker -- To question the unquestionable, ask the unaskable,
to think the unthinkable, mention the unmentionable, and challenge
the unchallengeable.

Re: If time goes slower for each twin

<JrudndxogdGzQV38nZ2dnUU7_8zNnZ2d@giganews.com>

  copy mid

https://www.novabbs.com/tech/article-flat.php?id=74997&group=sci.physics.relativity#74997

  copy link   Newsgroups: sci.physics.relativity
Path: i2pn2.org!i2pn.org!weretis.net!feeder6.news.weretis.net!news.misty.com!border2.nntp.dca1.giganews.com!nntp.giganews.com!buffer2.nntp.dca1.giganews.com!news.giganews.com.POSTED!not-for-mail
NNTP-Posting-Date: Mon, 20 Dec 2021 13:40:30 -0600
Date: Mon, 20 Dec 2021 13:40:29 -0600
MIME-Version: 1.0
User-Agent: Mozilla/5.0 (Macintosh; Intel Mac OS X 10.15; rv:91.0)
Gecko/20100101 Thunderbird/91.4.0
Subject: Re: If time goes slower for each twin
Content-Language: en-US
Newsgroups: sci.physics.relativity
References: <uwTWBSS2m6_vwEEBIi38LzTd43M@jntp>
<VuJDUZEFa_vj-RHzwrdu0oE2wvc@jntp> <sp4mus$78a$1@gioia.aioe.org>
<O90D20WUu984acPU1w_S90qqcuU@jntp> <4676671.GXAFRqVoOG@PointedEars.de>
<6MmCLEceSpwh8yfl41Jlt8j_tlA@jntp> <UXW1-e65QWxhFcysGp_wEQcIDvM@jntp>
<2089212.irdbgypaU6@PointedEars.de> <EQ3l7crtJC_TXLNyfniK_mjBZSo@jntp>
<12914471.uLZWGnKmhe@PointedEars.de> <RHSgLwi_4mlmyKU2Qx7KMAUkVkY@jntp>
<03e8378e-c4e4-469b-9316-15e3d9d5aab1n@googlegroups.com>
<v-Z96JM7oeo3z2tAH3OacbwhD2E@jntp> <sp7jnu$200$3@gioia.aioe.org>
<edb7909e-3fb6-40aa-b919-d164417c4383n@googlegroups.com>
<sp7mlj$1kmk$1@gioia.aioe.org>
<ef48c7ff-2589-4eba-8328-243a3402af47n@googlegroups.com>
<1pk4xfm.9ayruk1i0tv8vN%nospam@de-ster.demon.nl>
<8868015.CDJkKcVGEf@PointedEars.de>
<1pk6cuj.1mz43ke4p1ilN%nospam@de-ster.demon.nl>
<2223314.ElGaqSPkdT@PointedEars.de>
<1pk8wy0.1hwtxny6rumisN%nospam@de-ster.demon.nl>
<4c77b5bc-722f-4fc7-aae6-43e7663b1569n@googlegroups.com>
<1pkfhts.1dzlthd1upihk2N%nospam@de-ster.demon.nl>
From: tjrobert...@sbcglobal.net (Tom Roberts)
In-Reply-To: <1pkfhts.1dzlthd1upihk2N%nospam@de-ster.demon.nl>
Content-Type: text/plain; charset=UTF-8; format=flowed
Content-Transfer-Encoding: 7bit
Message-ID: <JrudndxogdGzQV38nZ2dnUU7_8zNnZ2d@giganews.com>
Lines: 28
X-Usenet-Provider: http://www.giganews.com
X-Trace: sv3-RdZ72pB8iCEun9YS7oTLm+FZmQBcL12+lcTWm0kTnRG22V4fZzKdFBzEGBiOHZkTQQIN772MdGV0egS!YoGJT/WXNGNOG7B9bUbUNWH+RBRNhXKFccL3vSmVLyyXP0LPJRWQ4ItgNrlNtWIz8EiFMfVLQA==
X-Complaints-To: abuse@giganews.com
X-DMCA-Notifications: http://www.giganews.com/info/dmca.html
X-Abuse-and-DMCA-Info: Please be sure to forward a copy of ALL headers
X-Abuse-and-DMCA-Info: Otherwise we will be unable to process your complaint properly
X-Postfilter: 1.3.40
X-Original-Bytes: 3466
 by: Tom Roberts - Mon, 20 Dec 2021 19:40 UTC

On 12/19/21 3:29 PM, J. J. Lodder wrote:
> Free falling observers see the same laws of physics,
> in their rest frame.

Interesting that you understand and accept this, but don't accept the
immediate corollary: identical clocks all tick at the same rate (in
their respective rest frames), if they are carried by free-falling
observers; this is independent of their location in a gravitational
potential, and of their relative velocities. After all, a clock's tick
rate is clearly determined by the laws of physics in its rest frame.

Actually your statement is not quite right, as gravity is excluded. A
better statement is:

The local laws of physics are the same in every locally
inertial frame.

(Gravity is due to the curvature of spacetime, and is thus not local.)

Note that for comparisons of atomic clock tick rates via EM signals, the
difference between free falling and located at rest on earth's surface
is negligible. This is so because the comparison takes such a very short
time that a free-falling frame initially equal to the earth-surface
frame diverges from the latter by a negligible amount during the
comparison. (Do not be deceived by a succession of comparisons that
together have a long duration.)

Tom Roberts

Re: If time goes slower for each twin

<1pkhrxo.uysbtn1a6girfN%nospam@de-ster.demon.nl>

  copy mid

https://www.novabbs.com/tech/article-flat.php?id=74999&group=sci.physics.relativity#74999

  copy link   Newsgroups: sci.physics.relativity
Path: i2pn2.org!i2pn.org!eternal-september.org!reader02.eternal-september.org!.POSTED!not-for-mail
From: nos...@de-ster.demon.nl (J. J. Lodder)
Newsgroups: sci.physics.relativity
Subject: Re: If time goes slower for each twin
Date: Mon, 20 Dec 2021 23:29:09 +0100
Organization: De Ster
Lines: 93
Message-ID: <1pkhrxo.uysbtn1a6girfN%nospam@de-ster.demon.nl>
References: <ade8707c-5779-44d2-9599-c42b4c2cdbf3n@googlegroups.com> <1pk4fg7.6nqt0j3osf61N%nospam@de-ster.demon.nl> <2088029.irdbgypaU6@PointedEars.de> <1pk7zpw.1u4709u12ez725N%nospam@de-ster.demon.nl> <jdadncq2e-PC5Cf8nZ2dnUU7_83NnZ2d@giganews.com> <1pk9vhw.133tjv1sel3x2N%nospam@de-ster.demon.nl> <ec11547f-e187-4a95-96f4-2bb1d4a309b4n@googlegroups.com> <1pkac7z.1frr3nrtaz7ilN%nospam@de-ster.demon.nl> <nK6dnY3H75YdDib8nZ2dnUU7_83NnZ2d@giganews.com> <1pkell2.1h3cfauu373o3N%nospam@de-ster.demon.nl> <86f37c21-821d-4d66-9e30-8f487644bb62n@googlegroups.com> <1pkfhui.3xn22u16ilj48N%nospam@de-ster.demon.nl> <68e8acd3-4e29-46a5-bdd6-75dc5f060cb8n@googlegroups.com>
Reply-To: jjlax32@xs4all.nl (J. J. Lodder)
Mime-Version: 1.0
Content-Type: text/plain; charset=ISO-8859-1
Content-Transfer-Encoding: 8bit
Injection-Info: reader02.eternal-september.org; posting-host="f60cbf13932189b385757d9f30b3e3d4";
logging-data="23247"; mail-complaints-to="abuse@eternal-september.org"; posting-account="U2FsdGVkX19hhqCbZWdiRifV01fuuvYzZGACodhJ2Ys="
User-Agent: MacSOUP/2.8.5 (ea919cf118) (Mac OS 10.10.5)
Cancel-Lock: sha1:VIN66DQmgT/zAEQCbdjYou9Weco=
 by: J. J. Lodder - Mon, 20 Dec 2021 22:29 UTC

Paparios <mrios@ing.puc.cl> wrote:

> El domingo, 19 de diciembre de 2021 a las 18:29:03 UTC-3, J. J. Lodder:
> > Paparios <mr...@ing.puc.cl> wrote:
> >
> > > El sábado, 18 de diciembre de 2021 a las 19:32:21 UTC-3, J. J. Lodder:
> > > > Tom Roberts <tjrobe...@sbcglobal.net> wrote:
> > > >
> > > > > On 12/16/21 8:42 AM, J. J. Lodder wrote:
> > > > > > Gravitational time dilatation is not a propagation effect.
> > > > >
> > > > > Hmmmm. See below.
> > > > >
> > > > > > It is inherent in the gravitational potential being higher or lower.
> > > > > > (yes, in Newtonian language)
> > > > >
> > > > > Hmmmm. See below.
> > > > You should stop Hmmm-Hmmming, and think and speak clearly.
> > > >
> > > > To make sure we talk about the same things I give a simple experiment.
> > > > Suppose we have a lab, and several optical clocks.
> > > > There is one on the lab bench, and suppose for convenience
> > > > that it ticks at 500 THz precisely, so at 5*10^14 Hz.
> > > > (with a stability of 10^-18)
> > > > Now take a second identical clock,
> > > > and place it on a pedestal about one meter higher on the bench.
> > > > So with a \Delta\Phi of 10 m^2/s^2. (with c^2 ~= 10^17)
> > > >
> > >
> > > That one meter difference in height, means both clocks are following
> > > different paths through spacetime.
> > While being both at rest in your room.
> > With sufficient purism you can even insist
> > that no two atoms can follow the same path through space-time.
> > That is just silly pedantry, without any empirical meaning.
> > > > By everybody's understanding of general relativity
> > > > (and perhaps also by yours)
> > > > it will tick at a rate of (1 + 10^-16) THz,
> > > > so at 5 x 10^14 + 0.05 Hz.
> > > > This difference in frequency is easily demonstrated.
> > > > Take light from the upper clock,
> > > > and bring it down with mirrors to the lower one,
> > > > and let them interfere.
> > > > You will see beats with a period of 20 seconds.
> > > > Bring light from the lower one up, and idem.
> > > > By direct frequency measurement using laser spectroscopy
> > > > you can establish that the higher clock is faster,
> > > > as predicted by general relativity.
> > >
> > > Sure, but in that case you are sending signals from the higher clock to
> > > the lower clock and then making the comparison there. In order to measure
> > > the time interval between them, the receiving clock must project that
> > > displacement 4-vector onto itself; that is a dot product involving the
> > > metric at the receiving clock's location.
> > No need to, but you snipped that.
> > In principle each clock can just count its ticks,
> > and you can see the counters diverge.
> >
> > Jan
>
> Actually, that is a much more complex task. One of the best tests on
> gravitational time dilation was performed using optical lattice clocks in
> Tokio. The experiment is decribed in the paper "Takamoto et al. (2020)
> Test of general relativity by a pair of transportable optical lattice
> clocks. Nat Photonics. doi: 10.1038/s41566-020-0619-8". You can read the
> paper at the following location:
> https://www.nature.com/articles/s41566-020-0619-8.pdf

Yes, doing it with portable clocks makes it more difficult.
Being Japanese it is not surprising that are interested.
They intend to set up a clock based monitoring network
to detect gravitational potential changes,
hence deep ground motions around volcanoes,
such as lighter material moving upwards.
Measuring differences in clock rates makes it possible to see that
even when the ground does not move. (yet)

BTW, the French have in the meantine set the next step:
they succeeded in building a van-mounted optical clock.
This will make it possible to quickly measure
clock frequency differences over a large distances,
between many fixed points.

> They need to check for around 10 centimeters height variations of the
> Tokio Tower (452 meters) due to temperature over the 6 months test period.
> Obviously, they are measuring the elapsed time of each clock and comparing
> their time difference to verify equation 1 in the paper.

Guess that is the equation for the difference in clock rates.
And yes, nice. The French should do the same for their Eiffel Tower.
It should even be possible to watch it cooling and shrinking overnight,

Jan

Re: If time goes slower for each twin

<1pki81d.1p0lzla4z2v8qN%nospam@de-ster.demon.nl>

  copy mid

https://www.novabbs.com/tech/article-flat.php?id=75001&group=sci.physics.relativity#75001

  copy link   Newsgroups: sci.physics.relativity
Path: i2pn2.org!i2pn.org!eternal-september.org!reader02.eternal-september.org!.POSTED!not-for-mail
From: nos...@de-ster.demon.nl (J. J. Lodder)
Newsgroups: sci.physics.relativity
Subject: Re: If time goes slower for each twin
Date: Mon, 20 Dec 2021 23:29:10 +0100
Organization: De Ster
Lines: 69
Message-ID: <1pki81d.1p0lzla4z2v8qN%nospam@de-ster.demon.nl>
References: <sp4mus$78a$1@gioia.aioe.org> <O90D20WUu984acPU1w_S90qqcuU@jntp> <4676671.GXAFRqVoOG@PointedEars.de> <6MmCLEceSpwh8yfl41Jlt8j_tlA@jntp> <UXW1-e65QWxhFcysGp_wEQcIDvM@jntp> <2089212.irdbgypaU6@PointedEars.de> <EQ3l7crtJC_TXLNyfniK_mjBZSo@jntp> <12914471.uLZWGnKmhe@PointedEars.de> <RHSgLwi_4mlmyKU2Qx7KMAUkVkY@jntp> <03e8378e-c4e4-469b-9316-15e3d9d5aab1n@googlegroups.com> <v-Z96JM7oeo3z2tAH3OacbwhD2E@jntp> <sp7jnu$200$3@gioia.aioe.org> <edb7909e-3fb6-40aa-b919-d164417c4383n@googlegroups.com> <sp7mlj$1kmk$1@gioia.aioe.org> <ef48c7ff-2589-4eba-8328-243a3402af47n@googlegroups.com> <1pk4xfm.9ayruk1i0tv8vN%nospam@de-ster.demon.nl> <8868015.CDJkKcVGEf@PointedEars.de> <1pk6cuj.1mz43ke4p1ilN%nospam@de-ster.demon.nl> <2223314.ElGaqSPkdT@PointedEars.de> <1pk8wy0.1hwtxny6rumisN%nospam@de-ster.demon.nl> <4c77b5bc-722f-4fc7-aae6-43e7663b1569n@googlegroups.com> <1pkfhts.1dzlthd1upihk2N%nospam@de-ster.demon.nl> <JrudndxogdGzQV38nZ2dnUU7_8zNnZ2d@giganews.com>
Reply-To: jjlax32@xs4all.nl (J. J. Lodder)
Injection-Info: reader02.eternal-september.org; posting-host="f60cbf13932189b385757d9f30b3e3d4";
logging-data="23247"; mail-complaints-to="abuse@eternal-september.org"; posting-account="U2FsdGVkX1+PjQUZuIJiDol2CoMzx0I3kZ3VDVDeT6I="
User-Agent: MacSOUP/2.8.5 (ea919cf118) (Mac OS 10.10.5)
Cancel-Lock: sha1:fbqfzVEncuvaopZP7/9DUC4KHGo=
 by: J. J. Lodder - Mon, 20 Dec 2021 22:29 UTC

Tom Roberts <tjroberts137@sbcglobal.net> wrote:

> On 12/19/21 3:29 PM, J. J. Lodder wrote:
> > Free falling observers see the same laws of physics,
> > in their rest frame.
>
> Interesting that you understand and accept this,

Have I ever said anything else?
FYI, you are being excessively silly wrt to me,
and you really should know better.

> but don't accept the
> immediate corollary: identical clocks all tick at the same rate (in
> their respective rest frames), if they are carried by free-falling
> observers; this is independent of their location in a gravitational
> potential, and of their relative velocities. After all, a clock's tick
> rate is clearly determined by the laws of physics in its rest frame.
>
> Actually your statement is not quite right, as gravity is excluded. A
> better statement is:
>
> The local laws of physics are the same in every locally
> inertial frame.
>
> (Gravity is due to the curvature of spacetime, and is thus not local.)

Yes, yes, technicalities.

> Note that for comparisons of atomic clock tick rates via EM signals, the
> difference between free falling and located at rest on earth's surface
> is negligible. This is so because the comparison takes such a very short
> time that a free-falling frame initially equal to the earth-surface
> frame diverges from the latter by a negligible amount during the
> comparison. (Do not be deceived by a succession of comparisons that
> together have a long duration.)

I'll try to make you see your mistake once more, in a simpler way.
We are dealing with two things here.
1) Real clocks, bolted to tables in laboratories, built and run by
experimenters who know their thing.
2) Non-existent clocks, flashing by on imagined world lines,
that only theoreticians can read in in their mind's eye.
You have not understood the connection between the two correctly.

Your imagined clocks may run at the same rate,
the real world experimenter's ones don't.
Any experimenter with state of the art optical clock will tell you that.
The differences are by now almost routinely measured,
and they have important technical applications.
It is by now a multi-million European research and development project,
in which about a dozen national metrology labs cooperate.

Since you haven't been to clear enough about how you think
that your imagined clocks and the real clocks in a laboratory
are related it is not possible for me to pinpoint your error.
OTOH, since your conclusions are so obviously counter-factual
you must have made a conceptual error somewhere.

BTW, I think you should get over it.
You made a mistake, and being as stubborn about it as the Woz
is doing you no good,

Jan

Re: If time goes slower for each twin

<8e469021-f58d-4021-badb-0ccd85430de8n@googlegroups.com>

  copy mid

https://www.novabbs.com/tech/article-flat.php?id=75004&group=sci.physics.relativity#75004

  copy link   Newsgroups: sci.physics.relativity
X-Received: by 2002:a05:622a:1112:: with SMTP id e18mr496043qty.226.1640046510789;
Mon, 20 Dec 2021 16:28:30 -0800 (PST)
X-Received: by 2002:a05:6214:1cc5:: with SMTP id g5mr609606qvd.92.1640046510625;
Mon, 20 Dec 2021 16:28:30 -0800 (PST)
Path: i2pn2.org!i2pn.org!weretis.net!feeder6.news.weretis.net!news.misty.com!border2.nntp.dca1.giganews.com!nntp.giganews.com!news-out.google.com!nntp.google.com!postnews.google.com!google-groups.googlegroups.com!not-for-mail
Newsgroups: sci.physics.relativity
Date: Mon, 20 Dec 2021 16:28:30 -0800 (PST)
In-Reply-To: <1pkfhts.1dzlthd1upihk2N%nospam@de-ster.demon.nl>
Injection-Info: google-groups.googlegroups.com; posting-host=205.154.192.197; posting-account=B_ql5woAAABEePt1fDMNH7lHfjEScu4Z
NNTP-Posting-Host: 205.154.192.197
References: <uwTWBSS2m6_vwEEBIi38LzTd43M@jntp> <sp3frr$itm$1@gioia.aioe.org>
<VuJDUZEFa_vj-RHzwrdu0oE2wvc@jntp> <sp4mus$78a$1@gioia.aioe.org>
<O90D20WUu984acPU1w_S90qqcuU@jntp> <4676671.GXAFRqVoOG@PointedEars.de>
<6MmCLEceSpwh8yfl41Jlt8j_tlA@jntp> <UXW1-e65QWxhFcysGp_wEQcIDvM@jntp>
<2089212.irdbgypaU6@PointedEars.de> <EQ3l7crtJC_TXLNyfniK_mjBZSo@jntp>
<12914471.uLZWGnKmhe@PointedEars.de> <RHSgLwi_4mlmyKU2Qx7KMAUkVkY@jntp>
<03e8378e-c4e4-469b-9316-15e3d9d5aab1n@googlegroups.com> <v-Z96JM7oeo3z2tAH3OacbwhD2E@jntp>
<sp7jnu$200$3@gioia.aioe.org> <edb7909e-3fb6-40aa-b919-d164417c4383n@googlegroups.com>
<sp7mlj$1kmk$1@gioia.aioe.org> <ef48c7ff-2589-4eba-8328-243a3402af47n@googlegroups.com>
<1pk4xfm.9ayruk1i0tv8vN%nospam@de-ster.demon.nl> <8868015.CDJkKcVGEf@PointedEars.de>
<1pk6cuj.1mz43ke4p1ilN%nospam@de-ster.demon.nl> <2223314.ElGaqSPkdT@PointedEars.de>
<1pk8wy0.1hwtxny6rumisN%nospam@de-ster.demon.nl> <4c77b5bc-722f-4fc7-aae6-43e7663b1569n@googlegroups.com>
<1pkfhts.1dzlthd1upihk2N%nospam@de-ster.demon.nl>
User-Agent: G2/1.0
MIME-Version: 1.0
Message-ID: <8e469021-f58d-4021-badb-0ccd85430de8n@googlegroups.com>
Subject: Re: If time goes slower for each twin
From: mark...@lycos.com (Mark-T)
Injection-Date: Tue, 21 Dec 2021 00:28:30 +0000
Content-Type: text/plain; charset="UTF-8"
Lines: 24
 by: Mark-T - Tue, 21 Dec 2021 00:28 UTC

On December 19, 2021 at 1:29:02 PM UTC-8, J. J. Lodder wrote:
>>>> But the reason is NOT that one clock is ticking faster than the other, but
>>>> that one clock measures *more elapsed proper time* than the other.
> > >
> >> There is nothing relative about it: clocks that stand higher run faster,
> >> objectively, absolutely, and for all observers.
>
> > Define "runs faster"
>
> Do you really need that?
> It runs faster if its ticks fall behind.

I'm trying to follow this chaotic debate, and have a basic question.

Given two clocks, in free fall, at different altitudes. They accumulate different
proper times. The higher clock "runs faster".

Is this difference invariant for all observers? Permit each clock (lab) to
broadcast its accumulated tick count, at each tick, digitally. Will all receivers,
whatever their state of motion, see the same rate of increasing discrepancy of
tick counts?

Mark

Re: If time goes slower for each twin

<375fd24d-473c-44d8-a8db-418a175220bdn@googlegroups.com>

  copy mid

https://www.novabbs.com/tech/article-flat.php?id=75012&group=sci.physics.relativity#75012

  copy link   Newsgroups: sci.physics.relativity
X-Received: by 2002:a05:6214:2409:: with SMTP id fv9mr1408014qvb.24.1640070880921;
Mon, 20 Dec 2021 23:14:40 -0800 (PST)
X-Received: by 2002:a05:620a:29d1:: with SMTP id s17mr1086238qkp.169.1640070880762;
Mon, 20 Dec 2021 23:14:40 -0800 (PST)
Path: i2pn2.org!i2pn.org!weretis.net!feeder6.news.weretis.net!news.misty.com!border2.nntp.dca1.giganews.com!nntp.giganews.com!news-out.google.com!nntp.google.com!postnews.google.com!google-groups.googlegroups.com!not-for-mail
Newsgroups: sci.physics.relativity
Date: Mon, 20 Dec 2021 23:14:40 -0800 (PST)
In-Reply-To: <1pkhrxo.uysbtn1a6girfN%nospam@de-ster.demon.nl>
Injection-Info: google-groups.googlegroups.com; posting-host=89.206.14.16; posting-account=I3DWzAoAAACOmZUdDcZ-C0PqAZGVsbW0
NNTP-Posting-Host: 89.206.14.16
References: <ade8707c-5779-44d2-9599-c42b4c2cdbf3n@googlegroups.com>
<1pk4fg7.6nqt0j3osf61N%nospam@de-ster.demon.nl> <2088029.irdbgypaU6@PointedEars.de>
<1pk7zpw.1u4709u12ez725N%nospam@de-ster.demon.nl> <jdadncq2e-PC5Cf8nZ2dnUU7_83NnZ2d@giganews.com>
<1pk9vhw.133tjv1sel3x2N%nospam@de-ster.demon.nl> <ec11547f-e187-4a95-96f4-2bb1d4a309b4n@googlegroups.com>
<1pkac7z.1frr3nrtaz7ilN%nospam@de-ster.demon.nl> <nK6dnY3H75YdDib8nZ2dnUU7_83NnZ2d@giganews.com>
<1pkell2.1h3cfauu373o3N%nospam@de-ster.demon.nl> <86f37c21-821d-4d66-9e30-8f487644bb62n@googlegroups.com>
<1pkfhui.3xn22u16ilj48N%nospam@de-ster.demon.nl> <68e8acd3-4e29-46a5-bdd6-75dc5f060cb8n@googlegroups.com>
<1pkhrxo.uysbtn1a6girfN%nospam@de-ster.demon.nl>
User-Agent: G2/1.0
MIME-Version: 1.0
Message-ID: <375fd24d-473c-44d8-a8db-418a175220bdn@googlegroups.com>
Subject: Re: If time goes slower for each twin
From: maluwozn...@gmail.com (Maciej Wozniak)
Injection-Date: Tue, 21 Dec 2021 07:14:40 +0000
Content-Type: text/plain; charset="UTF-8"
Content-Transfer-Encoding: quoted-printable
Lines: 96
 by: Maciej Wozniak - Tue, 21 Dec 2021 07:14 UTC

On Monday, 20 December 2021 at 23:29:12 UTC+1, J. J. Lodder wrote:
> Paparios <mr...@ing.puc.cl> wrote:
>
> > El domingo, 19 de diciembre de 2021 a las 18:29:03 UTC-3, J. J. Lodder:
> > > Paparios <mr...@ing.puc.cl> wrote:
> > >
> > > > El sábado, 18 de diciembre de 2021 a las 19:32:21 UTC-3, J. J. Lodder:
> > > > > Tom Roberts <tjrobe...@sbcglobal.net> wrote:
> > > > >
> > > > > > On 12/16/21 8:42 AM, J. J. Lodder wrote:
> > > > > > > Gravitational time dilatation is not a propagation effect.
> > > > > >
> > > > > > Hmmmm. See below.
> > > > > >
> > > > > > > It is inherent in the gravitational potential being higher or lower.
> > > > > > > (yes, in Newtonian language)
> > > > > >
> > > > > > Hmmmm. See below.
> > > > > You should stop Hmmm-Hmmming, and think and speak clearly.
> > > > >
> > > > > To make sure we talk about the same things I give a simple experiment.
> > > > > Suppose we have a lab, and several optical clocks.
> > > > > There is one on the lab bench, and suppose for convenience
> > > > > that it ticks at 500 THz precisely, so at 5*10^14 Hz.
> > > > > (with a stability of 10^-18)
> > > > > Now take a second identical clock,
> > > > > and place it on a pedestal about one meter higher on the bench.
> > > > > So with a \Delta\Phi of 10 m^2/s^2. (with c^2 ~= 10^17)
> > > > >
> > > >
> > > > That one meter difference in height, means both clocks are following
> > > > different paths through spacetime.
> > > While being both at rest in your room.
> > > With sufficient purism you can even insist
> > > that no two atoms can follow the same path through space-time.
> > > That is just silly pedantry, without any empirical meaning.
> > > > > By everybody's understanding of general relativity
> > > > > (and perhaps also by yours)
> > > > > it will tick at a rate of (1 + 10^-16) THz,
> > > > > so at 5 x 10^14 + 0.05 Hz.
> > > > > This difference in frequency is easily demonstrated.
> > > > > Take light from the upper clock,
> > > > > and bring it down with mirrors to the lower one,
> > > > > and let them interfere.
> > > > > You will see beats with a period of 20 seconds.
> > > > > Bring light from the lower one up, and idem.
> > > > > By direct frequency measurement using laser spectroscopy
> > > > > you can establish that the higher clock is faster,
> > > > > as predicted by general relativity.
> > > >
> > > > Sure, but in that case you are sending signals from the higher clock to
> > > > the lower clock and then making the comparison there. In order to measure
> > > > the time interval between them, the receiving clock must project that
> > > > displacement 4-vector onto itself; that is a dot product involving the
> > > > metric at the receiving clock's location.
> > > No need to, but you snipped that.
> > > In principle each clock can just count its ticks,
> > > and you can see the counters diverge.
> > >
> > > Jan
> >
> > Actually, that is a much more complex task. One of the best tests on
> > gravitational time dilation was performed using optical lattice clocks in
> > Tokio. The experiment is decribed in the paper "Takamoto et al. (2020)
> > Test of general relativity by a pair of transportable optical lattice
> > clocks. Nat Photonics. doi: 10.1038/s41566-020-0619-8". You can read the
> > paper at the following location:
> > https://www.nature.com/articles/s41566-020-0619-8.pdf
> Yes, doing it with portable clocks makes it more difficult.
> Being Japanese it is not surprising that are interested.
> They intend to set up a clock based monitoring network
> to detect gravitational potential changes,
> hence deep ground motions around volcanoes,
> such as lighter material moving upwards.
> Measuring differences in clock rates makes it possible to see that
> even when the ground does not move. (yet)

JJ, poor halfbrain, 2 clocks perfectly adjusted
to TAI may have different rates (frequencies, f),
but share the same indications (time, t). It's
either your time dilation idiocy or (xor) differences
in frequencies. According to your idiot guru TAI
is improper/wrong/evil, and that's what your fellow
idiot Tom is trying to explain you.


tech / sci.physics.relativity / Re: If time goes slower for each twin

Pages:12345678910111213141516
server_pubkey.txt

rocksolid light 0.9.81
clearnet tor